Secrets – Pediatric: Neurology

Secrets – Pediatric: Neurology

ANTIEPILEPTIC DRUGS
1. Should treatment with antiepileptic drugs (AEDs) be started after the first afebrile seizure in a child? Children with an isolated, uncomplicated seizure usually do not require AED therapy. Epidemiologic studies have shown that about one-third of children with an uncomplicated single seizure, a normal neurologic examination, and normal electroencephalogram (EEG) will experience a second. “Delaying” treatment until after the second seizure does not adversely affect the long-term chance of epilepsy remission. In fact, delaying treatment until 10 seizures may not affect remission, depending upon the underlying epilepsy syndrome.
AEDs are not without risks and side effects, both dose related and idiosyncratic. Other factors, including EEG results, antecedent neurologic history, family history, and imaging (in selective cases), influence the risk for recurrence and should be considered. Risk for recurrent seizures is sharply increased if the seizure was nocturnal, the neurologic status is not normal, there is a positive family history, if no immediate precipitating cause can be identified, and the EEG reveals epileptiform discharges. Not even status epilepticus as the initial seizure increases the overall risk of seizure recurrence, but does increase the risk that the next seizure could be status epilepticus.

Haut SR, Shinnar S: Considerations in the treatment of a first unprovoked seizure, Semin Neurol 3:289–296, 2008. Hirtz D, Berg A, Bettis D, et al, Quality Standards Subcommittee of the American Academy of Neurology; Practice Committee of the Child Neurology Society: Practice parameter: treatment of the child with a first unprovoked seizure. Report of the Quality Standards Subcommittee of the American Academy of Neurology and the Practice Committee of the Child Neurology Society, Neurology 60:166–172, 2003.

2. What is the advantage of monotherapy chosen according to the epilepsy syndrome?
• Chronic toxicity is directly related to the number of drugs consumed.
• As compared with monotherapy, intellectual and sensorium impairment is increased for any given AEDs (despite “normal” drug levels).
• Drug interactions may paradoxically lead to loss of seizure control.
• It is difficult to identify the cause of an adverse reaction.

Menkes JH, Sankar R: Paroxysmal disorders. In Menkes JH, Sarnat HB, Maria BL, editors: Child Neurology, ed 7, Philadelphia, 2006, Lippincott Williams & Wilkins, pp 891–893.

3. Which AEDs are recommended for primary generalized tonic-clonic seizures in children over 1 month of age? The “traditional” AEDs (phenobarbital, primidone, phenytoin) are no longer considered the drugs of choice for grand mal seizures for many age groups because of side effects. Studies have shown that most of the major anticonvulsants are comparable for reducing or eliminating seizure recurrences.
Class I evidence demonstrates that topiramate, lamotrigine, levetiracetam, valproate, and zonisamide are effective for the treatment of primary generalized tonic-clonic seizures. Carbamazepine and

501

oxcarbazepine may cause an increased frequency of primary generalized seizures, especially absence and myoclonic types, and are the drugs of choice for localization-related (focal) epilepsies.
Note that phenobarbital remains the first-line drug of choice for neonatal seizures.

Sankaraneni R, Lachhwani D: Antiepileptic drugs—a review, Pediatr Ann 44:e36–e42, 2015.
French JA, Kanner AM, Bautista J, et al: Therapeutics and Technology Assessment Subcommittee of the American Academy of Neurology; Quality Standards Subcommittee of the American Academy of Neurology; American Epilepsy Society: Efficacy and tolerability of the new antiepileptic drugs I: treatment of new onset epilepsy. Report of the Therapeutics and Technology Assessment Subcommittee and Quality Standards Subcommittee of the American Academy of Neurology and the American Epilepsy Society, Neurology 62:1252–1260, 2004.
Shankar R: Initial treatment of epilepsy with antiepileptic drugs: pediatric issues, Neurology 63:S30–S39, 2004.
4. What is the drug of choice for absence epilepsy?
Ethosuximide (Zarontin), valproate (divalproex sodium or Depakote), and lamotrigine (Lamictal) are all effective for eliminating or substantially reducing the number of absence attacks. Ethosuximide is traditionally the drug of choice, for several reasons:
• It works well for many patients. It not only stops the clinical attacks of absence, but it often normalizes the EEG by actually eliminating the 3/second spike-wave discharges.
• It is well tolerated by most patients. Although rare cases of serious bone marrow, liver, or dermatologic disorders have occurred, routine or frequent blood tests are not considered obligatory by most physicians.
• It has a relatively long serum half-life (40 hours). Thus, once- or twice-daily dosing is appropriate and represents a real convenience to the patient.
• It is relatively inexpensive.
A disadvantage is that ethosuximide protects only against absence seizures. Children with coexisting generalized convulsions should be treated with valproate or lamotrigine. Disadvantages of valproate include the risks for idiosyncratic liver toxicity, especially with an underlying metabolic
disease, pancreatopathy, thrombocytopenia (dose and duration-related), low vitamin D levels, osteopenia, weight gain, and teratogenicity. Lamotrigine should also be considered, with the relative
risk for rash (risk increases with valproic acid) and generally favorable cognitive profile taken into account.

Glauser TA, Cnaan A, Shinnar S, et al: Ethosuximide, valproic acid, and lamotrigine in childhood absence epilepsy, N Engl J Med 362:790–799, 2010.
5. Can AEDs paradoxically cause a worsening of seizures? A paradoxic worsening of seizure control by various AEDs has been noted for decades. In fact, every AED may aggravate seizures. Mechanisms may include nonspecific effects of drug intoxication. In addition, specific medications may exacerbate specific seizure types. For example, carbamazepine may worsen the absence, myoclonic, and astatic seizures seen in generalized epilepsy syndromes; phenytoin and vigabatrin may also worsen generalized seizures; and gabapentin and lamotrigine may worsen myoclonic seizures. The clinical trap to avoid is assuming that increasing seizures are related to the underlying epilepsy and that increasing doses are needed. With a paradoxic worsening, seizures will continue to worsen or not improve.

Perucca E, Gram L, Avanzini G, Dulac O: Antiepileptic drugs as a cause of worsening seizures, Epilepsia 39:5–17, 1998.
6. When should blood levels be obtained if seizures are poorly controlled, compliance is questionable, or drug toxicity suspected?
Trough serum drug levels should be obtained to detect subtherapeutic or toxic concentrations. It is most helpful to check the serum level right before the dose, preferably in the morning before any medication is given. An inadequate serum concentration is the most common cause of persistent seizures, but drug toxicity, especially with phenytoin, may also manifest by deteriorating seizure control. There generally will be less variation in blood concentrations with tablets or capsules compared with liquid preparations; suspensions in particular result in notoriously inconsistent dosages. If drug toxicity is suspected, peak serum levels may be preferable to trough serum levels.

Stepanova D, Beran RG: The benefits of antiepileptic drug (AED) blood level monitoring to complement clinical management of people with epilepsy, Epilepsy BehAV 42:7–9, 2015.
7. What are the suggested therapeutic ranges of AEDs?
See Table 13-1.

Table 13-1. Drug/Trade Name Target Plasma Drug Concentration Range*

DRUG (GENERIC) BRAND NAME TARGET DRUG LEVEL
Carbamazepine Tegretol/Carbatrol 4-12 mcg/mL
Clobazam Onfi Not done
Ethosuximide Zarontin 40-100 μg/mL
Felbamate Felbatol 30-100 μg/mL
Gabapentin Neurontin 4-20 μg/mL
Lacosamide Vimpat 5-10 μg/mL
Lamotrigine Lamictal 2-20 mcg/mL
Levetiracetam Keppra 10-60 μg/mL
Oxcarbazepine Trileptal 10-40 μg/mL
Phenobarbital Luminal 10-45 μg/mL
Phenytoin Dilantin, Fosphenytoin 10-20 mcg/mL
Primidone Mysoline 5-10 mg/mL
Rufinamide Banzel 3-30 μg/mL
Tiagabine Gabitril 20-200 ng/mL
Topiramate Topamax 2-25 μg/mL
Valproate Depakene/Depakote 50-100 mcg/mL
Vigabatrin Sabril Not done
Zonisamide Zonegran 10-40 μg/mL
*Therapeutic ranges are just guidelines for dosing. Seizures may respond to a low level, and toxicity may occur at a low level. Levels above “normal” may be needed to control seizures and can be used, if tolerated. It is important to treat the patient and not the “level.”

8. What are the typical dose-related side effects of AEDs?
Dose-related side effects occur somewhat predictably and can be anticipated, particularly as the medication dose is initiated and escalated. Common dose-related side effects include sedation, headache, gastrointestinal irritation, unsteadiness, and dysarthria. Management commonly consists of reducing the dose by 25% to 50% and waiting about 2 weeks for tolerance to develop. In addition, behavioral and cognitive side effects can occur in some patients; these can be subtler, and controversy exists regarding the relative effects of various AEDs.
Dose-related side effects are referred to as “nuisance” side effects, which typically resolve either with dose-reduction or tolerance, compared with direct organ toxicity (liver, pancreas, bone marrow).

Guerrini R, Zaccara G, Ia Marca G, Rosati A: Safety and tolerability of antiepileptic drug treatment in children with epilepsy,
Drug Saf 35: 519–533, 2012.

9. What idiosyncratic drug reactions are associated with antiepileptic medications?
Idiosyncratic reactions occur unpredictably, are potentially fatal, and do not correlate with dose of medication.
• Carbamazepine: leukopenia, aplastic anemia, thrombocytopenia, hepatic dysfunction, rashes
• Ethosuximide: leukopenia, pancytopenia, rashes
• Phenobarbital: rashes, Stevens-Johnson syndrome, hepatic dysfunction
• Phenytoin: hepatic dysfunction, lymphadenopathy, movement disorder, Stevens-Johnson syndrome, fulminant hepatic failure
• Valproic acid: fulminant hepatic failure (especially in at-risk patients, see question 10), hyperammonemia, pancreatitis, thrombocytopenia, rash, stupor
• Lamotrigine: stevens-Johnson syndrome, toxic epidermal necrolysis, hepatic failure, DRESS syndrome (Drug Rash with Eosinophilia and Systemic Symptoms), aseptic meningitis

10. Which children are most susceptible to valproic acid–induced acute hepatic failure? The highest incidences occur in children younger than 2 years who are receiving polytherapy (1 in 540). In children younger than 2 years who are receiving valproic acid monotherapy, the rate is reduced to about 1 in 8000. The complication is unrelated to dosage and typically occurs during the first 3 months of therapy. Up to 40% of individuals who receive valproic acid will have dose-related elevations of liver enzymes that are transient or resolve with dosage adjustments. However, liver function test monitoring is not helpful for predicting acute hepatic failure. It has been hypothesized that valproic acid may cause carnitine deficiency, hyperammonemia, and hepatotoxicity. Despite a lack of data from clinical trials, some clinicians recommend prophylactic carnitine supplementation.
In addition, Alpers syndrome is a rare, autosomal recessive neurometabolic disease, often presenting with intractable seizures and psychomotor regression. The disease is caused by a mutation in POLG, which encodes for a mitochondrial DNA polymerase. Children with Alpers syndrome are at increased risk for fulminant hepatic failure with valproic acid administration, and this agent should be used with caution if this condition is suspected.

Saneto RP, Lee IC, Koenig MK, et al: POLG DNA testing as an emerging standard of care before instituting valproic acid therapy for pediatric seizure disorders, Seizure 19:140–146, 2010.
Bryant AE, Dreifuss FE: Valproic acid hepatic fatalities, Neurology 48:465–469, 1996.

11. What are the warning signs and symptoms of hypersensitivity syndromes to AEDs? Symptoms often occur early, within the first months of treatment. Families need to be educated about the potential for drug reactions. Concerning symptoms include:
• body temperature higher than 40 °C
• protracted vomiting
• lethargy
• exfoliation of the skin (palm or sole) or mucosal lesions
• facial edema or swelling of the tongue
• confluent erythema, palpable purpura
• protracted bleeding from minor cuts
• lymph node enlargement
• wheezing (indicating anaphylaxis)
Multiple studies have shown that families fail to appreciate evolving symptoms of idiosyncratic reactions and continue to administer the offending agent. Laboratory abnormalities may include eosinophilia, atypical lymphocytosis, and abnormal liver function enzymes. Routine surveillance of blood chemistries and complete blood counts (every 3 to 6 months) is standard practice, but they are unlikely to identify potentially life-threatening conditions.

Ye YM, Thong BY, Park HS: Hypersensitivity to antiepileptic drugs, Immunol Allergy Clin North Am 34:633–643, 2014.

12. What is Diastat?
Diazepam rectal gel (Diastat) has been approved for the treatment of status epilepticus and severe recurrent convulsive seizures in children. It is prescribed for home use by parents as an emergency medication. Dosages (as they are for most medications in pediatric patients) are based on weight, and the medication is available in various premixed concentrations with syringe applicators. Parents are generally counseled to administer the medication for a seizure lasting greater than 5 minutes and to call 911 with administration because respiratory depression can occur with administration of benzodiazepines.
13. Are there other options for outpatient treatment of severe recurrent and prolonged seizures in children?
As of 2014, rectal diazepam is the only formulation approved in the United States by the FDA for out-of- hospital treatment. However, clinical trials are in progress for alternatives such as intranasal midazolam, diazepam and lorazepam, buccal midazolam (approved in the European Union), sublingual lorazepam, and intramuscular diazepam by autoinjection.

McKee HR, Abou-Khalil B: Outpatient pharmacotherapy and modes of administration for acute repetitive and prolonged seizures, CNS Drugs 29:55–70, 2015.

14. After what period can AEDs be safely discontinued? The withdrawal of AEDs should be considered when the child is free of seizures for 2 years because well- controlled investigations have shown that the risk for relapse in children whose seizures have been in remission for 2 years is low. A 4-year period of seizure freedom was the previous standard. Although there is no uniform agreement about factors that are predictive of outcome, the highest remission rate appears to occur in those who are otherwise neurologically normal and in whom the EEG at the time of discontinuation lacks specific epileptiform features and displays a normal background. The prognosis is the worst for children with symptomatic epilepsies, persistently abnormal EEGs, and abnormal neurologic examinations. Remission also depends upon the underlying epilepsy syndrome.

Camfield P, Camfield C: When is it safe to discontinue AED treatment? Epilepsia 49S:25–28, 2008. Smith R, Ball R: Discontinuing anticonvulsant medication in children, Arch Dis Child 87:259–260, 2002.

15. When the decision is made to discontinue AEDs, should the tapering period be long or short?
In practice, all AEDs should be tapered gradually rather than abruptly discontinued, although there is no actual withdrawal state produced by a “cold turkey” reduction of most AEDs (e.g., phenytoin, carbamazepine, valproate, ethosuximide). By contrast, a withdrawal syndrome of agitation, signs of autonomic overactivity, and seizures follows the sudden elimination of habitually consumed benzodiazepines (diazepam, clonazepam, clobazam, lorazepam) or short-acting barbiturates (e.g., secobarbital). The long elimination half-life of phenobarbital lessens the risk for withdrawal symptoms after abrupt discontinuation.
In a study of more than 100 children who had been seizure free for either 2 or 4 years, the risk for seizure recurrence during tapering and after discontinuation of the AED was no different if the period of taper was 6 weeks or 9 months. Rapid tapering appears to be an acceptable means of discontinuation.

Tennison M, Greenwood R, Lewis D, Thorn M: Discontinuing antiepileptic drugs in children with epilepsy: a comparison of a six-week and a nine-month taper period, N Engl J Med 330:1407–1410, 1994.

CEREBRAL PALSY
16. What is cerebral palsy?
Cerebral palsy (CP) describes a heterogeneous group of nonprogressive (static) motor and posture disorders of cerebral or cerebellar origin that typically manifest early in life. The primary impairment involves significant deficits in motor planning and control. Nonprogressive, clinical manifestations often change over time as the functional expression of the underlying brain is modified by brain development and maturation. However, motor function that is affected results from the part of the brain that is injured. Causes include cerebral malformations, metabolic and genetic causes, infection (both intrauterine and extrauterine), stroke, hypoxia-ischemia, and trauma.

American Academy for Cerebral Palsy and Developmental Medicine: www.aacpdm.org. Accessed on March 5, 2015. United Cerebral Palsy Association: www.ucp.org. Accessed on March 5, 2015.

17. What are the most common brain lesions seen on magnetic resonance imaging (MRI) in children with cerebral palsy? Periventricular white matter lesions are the most common and can be seen in 19% to 45% of children with CP (particularly formerly premature infants). Other common lesions include gray matter injuries of the basal ganglia and thalamus (21%), developmental cortical malformations (11%), and focal cortical infarcts (10%). Up to 15% of cases of CP do not have an identifiable lesion on MRI. The varied MRI findings are believed to be emblematic of the neurodevelopmental heterogeneity of CP.

Hadders-Algra M: Early diagnosis and early intervention in cerebral palsy, Front Neurol 5:185, 2014.

18. What are the Levine (POSTER) criteria for the diagnosis of CP?
• Posturing/abnormal movements
• Oropharyngeal problems (e.g., tongue thrusts, swallowing abnormalities)

• Strabismus
• Tone (hypertonia or hypotonia)
• Evolutional maldevelopment (primitive reflexes persist or protective/equilibrium reflexes fail to develop [e.g., lateral prop, parachute reflex])
• Reflexes (increased deep tendon reflexes/persistent Babinski reflex)
Abnormalities in four of these six categories strongly point to the diagnosis of CP.

Feldman HM: Developmental-behavioral pediatrics. In Zitelli BJ, Davis HW, editors: Atlas of Pediatric Diagnosis, ed 5, St. Louis, 2007, Mosby, p 82.

19. What are the types of cerebral palsy?
Clinical classification is based on the nature of the movement disorder and muscle tone and anatomic distribution. A single patient may have more than one type. Spastic cerebral palsy is the most common, accounting for about two-thirds of cases.
Spastic (or pyramidal) CP: Characterized by neurologic signs of upper motor neuron damage with increased “clasp knife” muscle tone, increased deep tendon reflexes, pathologic reflexes, and spastic weakness. Spastic CP is subclassified based on distribution:
• Hemiplegia: Primarily unilateral involvement, arm usually more than leg
• Quadriplegia: All limbs involved, with legs often more involved than arms
• Diplegia: Legs much more involved than arms, which may show no or only minimal impairment (more common in the premature infant)
Dyskinetic (nonspastic or extrapyramidal) CP: Characterized by prominent involuntary movements or fluctuating muscle tone, with choreoathetosis the most common subtype. Distribution is usually symmetric among the four limbs.
Ataxic CP: Primarily cerebellar signs (including ataxia, dysmetria, past pointing, nystagmus)
Mixed types: Features of multiple types of cerebral palsy

Richards CL, Malouin F: Cerebral palsy: definition, assessment and rehabilitation, Handb Clin Neurol 111: 183–195, 2013.
Murphy N, Such-Neibar T: Cerebral palsy diagnosis and management: the state of the art, Curr Probl Pediatr Adolesc Health Care 33:146–169, 2003.

20. What proportion of CP is related to birth asphyxia?
In contrast with popular perception, large clinical epidemiologic and longitudinal studies indicate that perinatal asphyxia is an important—but relatively minor—cause. Estimates range from a low of 3% to a high of 21%. In most cases, the events leading to CP occur in the fetus before the onset of labor or in the newborn after delivery.

McIntyre S, Blair E, Badawi N, et al: Antecedents of cerebral palsy and perinatal death in term and late preterm singletons,
Obstet Gynecol 122:869–877, 2013.
Nelson KB: Can we prevent cerebral palsy? N Engl J Med 349:1765–1769, 2003.

21. How well do Apgar scores correlate with the development of CP?
Large studies have mixed results. A 1981 study of 49,000 infants found that a low Apgar score correlated poorly with the development of CP. Of term infants with scores of 0 to 3 at 1 or 5 minutes, 95% did not develop CP. Conversely, nearly 75% of patients with CP had 5-minute Apgar scores of 7 to
10. More recent studies have found a stronger association between low Apgar scores and cerebral palsy in term infants, but there is no clear association for low birth weight or premature infants.
A 2010 population study of over 500,000 Norwegian infants found an association between an Apgar score <4 at 5 minutes and cerebral palsy, which was strongest for infants with normal birth weight and for patients later diagnosed with quadriplegia.

Lie KK, Groholt E-K, Eskild A: Association of cerebral palsy with Apgar score in low and normal birthweight infants: population based cohort study, BMJ 341:c4990, 2010.
Nelson KB, Ellenberg JH: Apgar scores as predictors of chronic neurologic disability, Pediatrics 68:36–44, 1981.

22. Why is CP difficult to diagnose clinically during the first year of life? Unlike adults with acute neurologic deficits, which may be focal, young children may manifest generalized and nonspecific neurologic dysfunction following an acute neurologic insult:
• Hypotonia is more common than hypertonia in the first year following an acute insult and spasticity typically develops later, both of which makes the prediction of CP difficult. Hypertonia, especially in the antigravity muscles, develops to compensate for weakness. Initial hypertonia may be seen with a basal ganglia insult.
• The early abundance of primitive reflexes (with variable persistence) may confuse the clinical picture.
• An infant has a limited variety of volitional movements for evaluation.
• Substantial myelination takes months to evolve and may delay the clinical picture of abnormal tone and increased deep tendon reflexes.
• Most infants who develop CP do not have identifiable risk factors; most cases are not related to labor and delivery events (intrapartum).
Most cases of cerebral palsy can be diagnosed by 18 to 24 months of life.

Hadders-Algra M: Early diagnosis and early intervention in cerebral palsy, Front Neurol 5:185, 2014.
Shapiro BK, Capute AJ: Cerebral palsy. In McMillan JA, DeAngelis CD, Feigin RD, Warshaw JB, editors: Oski’s Pediatrics, Principles and Practice, ed 3, Philadelphia, 1999, Lippincott Williams & Wilkins, pp 1910–1917.

KEY POINTS: CEREBRAL PALSY
1. Apgar scores correlate relatively poorly with the ultimate diagnosis of cerebral palsy.
2. During the first year of life, hypotonia is more common than hypertonia in patients who are ultimately diagnosed with the disease.
3. Keep an eye on the eyes: As many as 75% of children with cerebral palsy have ophthalmologic problems (e.g., strabismus, refractive errors).
4. Spastic hemiplegia is the most common type of cerebral palsy that is associated with seizures.
5. Monitor regularly for hip subluxation, especially in patients with spastic diparesis, because earlier identification assists therapy.

23. What behavioral symptoms during the first year should arouse suspicion about the possibility of CP?
• Excessive irritability, constant crying, and sleeping difficulties (severe colic is noted in up to 30% of babies who are eventually diagnosed with CP)
• Early feeding difficulties with difficulties in coordinating suck and swallow, frequent spitting up, and poor weight gain
• “Jittery” or “jumpy” behavior, especially at times other than when hungry
• Easily startled behavior
• Stiffness when handled, especially during dressing, diapering, and handwashing
• Paradoxically “precocious” development, such as early rolling (actually a sudden, reflexive roll rather than a volitional one) or apparent early strength, such as the stiff-legged “standing” with support of an infant with spastic diplegia

Bennett FC: Diagnosing cerebral palsy—the earlier the better, Contemp Pediatr 16:65–76, 1999.

24. What gross motor delays are diagnostically important in the infant with possible CP?
• Inability to bring the hands together in midline while in a supine position by the age of 4 months
• Head lag persisting beyond 6 months
• No volitional rolling by 6 months
• Inability to independently sit straight by 8 months
• No hands-and-knees crawling by 12 months

Bennett FC: Diagnosing cerebral palsy—the earlier the better, Contemp Pediatr 16:65–76, 1999.

25. What problems are commonly associated with CP?
• Mental retardation: Two-thirds of total patients; most commonly observed in children with spastic quadriplegia
• Failure to thrive, growth retardation
• Feeding problems (including dysphagia, sialorrhea [excessive salivation])
• Gastrointestinal problems (gastroesophageal reflux, constipation)
• Learning disabilities
• Ophthalmologic abnormalities (strabismus, amblyopia, nystagmus, refractive errors)
• Hearing deficits
• Communication disorders
• Epilepsy: One-half of total patients; most commonly observed in children with spastic hemiplegia and related to the degree of neuroimaging abnormality.
• Behavioral and emotional problems (especially attention-deficit hyperactivity disorder, depression, sleep problems)
• Urinary problems (incontinence, voiding dysfunction, urinary tract infections)
• Spinal column changes (kyphosis, scoliosis)
• Respiratory problems (upper airway obstruction, chronic aspiration)

Sewell MD, Eastwood DM, Wimalasundera N: Managing common symptoms of cerebral palsy in children, BMJ 349: g4596, 2014.
Dodge NN: Cerebral palsy: medical aspects, Pediatr Clin North Am 55:1189–1207, 2008.

26. What features in an infant suggest a progressive central nervous system (CNS) disorder rather than CP as the cause of a motor deficit?
Abnormally increasing head circumference: Possible hydrocephalus, tumor, or neurodegenerative disorder
Eye anomalies: Cataracts, retinal pigmentary degeneration, optic atrophy (possible neurodegenerative disease), coloboma, chorioretinal lacuna, optic nerve hypoplasia (possible Aicardi syndrome or septo-optic dysplasia)
Skin abnormalities: Vitiligo, café-au-lait spots, nevus flammeus, port-wine stain (possible Sturge-Weber syndrome or neurofibromatosis)
Hepatomegaly and/or splenomegaly (possible metabolic or lysosomal storage disease)
Decreased or absent deep tendon reflexes
Sensory abnormalities: Diminished sense of pain, position, vibration, or light touch
Developmental regression or failure to progress: Rett syndrome or Leigh disease

Taft LT: Cerebral palsy, Pediatr REV 16:411–418, 1995.

27. What therapies are used to treat the spasticity and dystonia of cerebral palsy? Casting: Serial “inhibitive” casting can reduce tone and allow improved gait and weight-bearing activities Nerve blocks, motor point blocks, botulinum toxin: Injected to target spasticity in particular muscle
groups
Oral and intrathecal medications: Including baclofen, dantrolene, carbidopa-levodopa, clonazepam Tendon-lengthening surgeries: At ankle, knee, wrist, or elbow to prevent or delay joint contractures Selective dorsal rhizotomy: A neurosurgical procedure that interrupts the afferent component of the
deep tendon (stretch) reflex

Ailon T, Beauchamp R, Miller S, et al: Long-term outcome after selective dorsal rhizotomy in children with spastic cerebral palsy, Childs NERV Syst 31: 415-423, 2015.
Copeland L, Edwards P, Thorley M, et al: Botulinum toxin A for nonambulatory children with cerebral palsy: a double-blind randomized controlled trial, J Pediatr 165:140–146, 2014.

CEREBROSPINAL FLUID
28. What is normal cerebrospinal fluid (CSF) pressure?
CSF pressure as measured during a lumbar puncture varies with age, positional technique, and combativeness of the patient. For truly accurate pressures, the child should be relaxed with legs extended.

CSF can be seen in the manometer varying with respirations when the needle has been properly placed. As a general guide, the normal ranges of CSF opening pressures are:
• Neonate: 80 to 100 mm H2O /CSF
• 1 month to 4 years: 10 to 100 mm H2O
• 8 years to adolescent-adult: 100 to 200 mm H2O
• Adolescent to adult: 100 to 250 mm H2O (may reach 280 in obese or sedated patients). Any opening pressure >250 mm H2O should be considered suspicious for intracranial hypertension.
Some evidence exists that in the obese or sedated patient, a cutoff of 280 mm H2O may be
considered the upper limit of normal. Both elevated body mass index (BMI) and deep sedation may increase the opening pressure.

Avery RA: Interpretation of lumbar puncture opening pressure measurements in children, J Neuroophthalmol 34: 284–287, 2014.
Ellis R 3rd: Lumbar cerebrospinal fluid opening pressure measured in flexed lateral decubitus position in children,
Pediatrics 93:622–623, 2003.

29. What are the normal CSF volumes in an infant, child, and adolescent?
Estimates for the volume of the ventricular system are:
• 40 to 50 mL in a term newborn
• 65 to 100 mL in an older child
• 90 to 150 mL in a teenager or adult The choroid plexus actively secretes a distillate of CSF at a rate of 0.3 to 0.4 mL/minute in children
and adults, which equals about 20 mL/hour or 500 mL/day. This equates to an hourly CSF volume turnover rate of about 15%.
30. What are the common causes of an elevated CSF protein?
Elevated CSF protein (>30 mg/dL) is a nonspecific finding that is encountered in various neurologic disorders. Several common etiologies should be considered:
Infection: Tuberculous meningitis, acute bacterial meningitis (pneumococcal, meningococcal,
Haemophilus influenzae), syphilitic or viral meningitis, encephalitis
Inflammation: Guillain-Barré syndrome (GBS), multiple sclerosis, peripheral neuropathy, postinfectious encephalopathy
Tumor of the cerebral hemispheres or spinal cord; A CSF block may cause a very elevated CSF protein.
Vascular accidents, such as cerebral hemorrhage (including subarachnoid hemorrhage, subdural hemorrhage, intracerebral hemorrhages) or stroke as a result of cranial arteritis, diabetes mellitus, or hypertension
Degenerative disorders involving white-matter disease (e.g., Krabbe disease)
Metabolic disorders (e.g., uremia)
Toxins (e.g., lead)
Prematurity, related to immaturity of the blood-brain barrier

31. What CSF findings suggest metabolic disease as a cause of neurologic signs and symptoms?
• Elevated CSF protein concentration is characteristic of metachromatic leukodystrophy and globoid cell encephalopathy.
• Low CSF glucose concentration is consistent with hypoglycemia caused by a defect of gluconeogenesis or a defect in the transport of glucose across the blood-brain barrier (GLUT-1 deficiency syndrome).
• Low CSF folate concentration suggests a defect involving folate metabolism.
• Presence in the CSF of amino acids, specifically glycine, glutamate, and γ-aminobutyric acid (GABA), may be diagnostic of nonketotic hyperglycinemia, pyridoxine-dependent epilepsy, or another defect in GABA metabolism.
• Lactate and pyruvate values are elevated in CSF disorders of cerebral energy metabolism, including pyruvate dehydrogenase deficiency, pyruvate carboxylase deficiency, numerous disturbances of the respiratory chain, and Menkes syndrome.
• Low CSF lactate value may be seen in the GLUT-1 deficiency syndrome.
• Abnormal CSF biogenic amines suggest several disorders that are associated with disturbed neurotransmission.

32. Why is a stylet used during a lumbar puncture?
A stylet is typically used during a lumbar puncture to prevent epidermis (which might lodge in an open- ended needle) from being introduced into the subarachnoid space, where an epidermal tumor might form. There is debate about whether the stylet should be kept in place after the needle passes the subcutaneous space or removed at that point to allow a better assessment of CSF flow when the needle enters the subarachnoid space. After fluid has been collected, some experts advocate reinserting the stylet to minimize the potential to prevent attached arachnoid strands from causing prolonged CSF leakage through the dura, which may cause prolonged headaches. In newborns, it has been more common to not use a stylet, although this practice should be avoided.

Ellenby MS, Tegtmeyer K, Lai S, Braner DAV: Videos in clinical medicine. Lumbar puncture, N Engl J Med 355:e12:2006. Baxter AL, Fisher RG, Burke BL, et al: Local anesthetic in stylet styles: factors associated with resident lumbar puncture success, Pediatrics 117:876–881, 2006.
Strupp M, Brandt T, Muller A: Incidence of post-lumbar puncture syndrome reduced by reinserting the stylet: a randomized prospective study of 600 patients, J Neurol 245:589–592, 1998.

33. As tests of meningeal irritation, what constitutes a positive Kernig or Brudzinski sign?
• Kernig sign, or the straight-leg-raising sign, consists of flexing the hip to 90 degrees and attempting to extend the knee. The limitation of knee extension as a result of painful resistance is a positive sign.
• Brudzinski sign is a positive sign, and it is present if a reflex flexion of the thighs occurs when a patient’s neck is passively flexed.
It should be noted that these signs may not be present in the infant or young child (<18 to 24 months).
34. How do the manifestations of increased intracranial pressure (ICP) differ in an infant compared with an older child?
• Infant: Increasing head circumference, delayed closure of the fontanel, suture separation, bulging fontanel, failure to thrive, macrocephaly, paresis of upward gaze (known as setting-sun sign) shrill cry
• Older child: Headache (especially in the early morning, awakening the child from sleep, or association with vomiting), nausea, persistent vomiting, personality and mood changes, lethargy, anorexia, fatigue, somnolence, diplopia as a result of sixth-nerve palsy or third-nerve palsy with uncal herniation, papilledema (Fig. 13-1)

Figure 13-1. Papilledema. Signs include swelling of the optic disc with blurring of normally sharp margins, venous engorgement, and curvature of blood vessels due to elevation of the disc. (From Douglas G, Nicol F, Robertson C, editors: MacLeod’s Clinical Examination, ed 13. London, 2013, ELSEVIER, Ltd.,
p 285.)

35. What comprises the Cushing triad? The Cushing triad consists of the development of slow or irregular respirations, decreased heart rate, and elevated blood pressure (particularly an increased systolic pressure with a widening pulse pressure) resulting from an increase in ICP. The Cushing triad may be observed in children with increased ICP or compression of the posterior fossa, which houses the medullary circulatory control center. It is a very late finding of increased ICP.

36. How is hydrocephalus classified?
Communicating hydrocephalus is caused by an inability to normally reabsorb CSF by the arachnoid granulations, which can occur from meningeal scarring as a result of bacterial meningitis, intraventricular hemorrhage, or intrathecal chemotherapy. It can be diagnosed if a tracer dye injected into one lateral ventricle appears in the lumbar CSF.
Noncommunicating hydrocephalus refers to conditions causing intraventricular obstruction and alteration of the flow of dye into the lumbar CSF. Congenital malformations (especially aqueductal stenosis and Dandy-Walker syndrome with cystic dilation of the fourth ventricle) and mass lesions (e.g., tumors, arteriovenous malformations) can cause noncommunicating hydrocephalus.
Hydrocephalus ex VACUO describes increases in CSF volume without increased CSF pressure, which is seen in conditions of reduced cerebral tissue (e.g., malformation, atrophy).

37. What is the normal growth rate of head circumference during the first year of life? Head circumference at birth is about 34 to 35 cm for the term infant. The head circumference normally grows by 2 cm/month for the first 3 months of life, 1 cm/month for months 4 to 6, and 0.5 cm/month up to 1 year of life. The measurement of head circumference should be part of the examination of any child and should be plotted at every visit. The head circumference represents brain growth, but it is also influenced by hydrocephalus and subdural or epidural fluid collections.
38. What are the complications of ventricular shunts? Ventricular shunts drain CSF from the ventricles in patients whose normal outflow or absorption has been blocked. The fluid may be drained to a variety of different locations, including the peritoneum, kidney, or cardiac atrium. Shunts draining CSF have remarkably improved the outcome of children with hydrocephalus, but they are subject to obstruction, infection, or mechanical malfunction. Shunt malfunctions present with signs of increased ICP. Children with shunt infections may have a low-grade fever, as well as signs of increased ICP. Because it is impossible to know the compliance properties of the ventricular system, children with shunt malfunction or infection are at risk for sudden, catastrophic decompensation. Children suspected of having shunt malfunctions or infection require urgent attention, and they should be closely observed until the shunt has been fully evaluated.

Rogers EA, Kimia A, Madsen JR, et al: Predictors of ventricular shunt infection among children presenting to a pediatric emergency department, Pediatr Emerg Care 28:405–409, 2012.
Piatt JH Jr, Garton HJL: Clinical diagnosis of ventriculoperitoneal shunt failure among children with hydrocephalus, Pediatr Emerg Care 24:210–210, 2008.

39. What are the characteristic features of idiopathic intracranial hypertension? Idiopathic intracranial hypertension consists of increased ICP in the absence of a demonstrable mass lesion and with a normal CSF formula. The condition was formerly called pseudotumor cerebri or benign intracranial hypertension. The term “benign” has been de-emphasized because the problem has the potential to cause significant visual loss and to disrupt normal activities of daily living. Characteristic features include the following:
• Headache, fatigue, vomiting, anorexia, stiff neck, and diplopia from increased ICP
• Normal neurologic examination except for papilledema or a third- or sixth-nerve palsy
• Visual field constriction (usually nasal field) and enlargement of the blind spot on confrontational testing
• Normal computed tomography scan, except sometimes for small ventricles
• Normal CSF profile with the exception of an elevated opening pressure >250 mm H2O

Glatstein MM, Oren A, Amarilyio G, et al: Clinical characterization of idiopathic intracranial hypertension in children presenting to the emergency department: the experience of a large tertiary care pediatric hospital, Pediatr Emerg Care 31:6–9, 2015.
Krishnakumar D, Pickard JD, Czosnyka Z, et al: Idiopathic intracranial hypertension in childhood: pitfalls in diagnosis, DEV Med Child Neurol 56:749–755, 2014.

40. What causes idiopathic intracranial hypertension? Although there are multiple possible causes, more than 90% of cases are idiopathic. Among the reported causes are the following:
• Drugs: Tetracycline, nalidixic acid, nitrofurantoin, corticosteroids, excess vitamin A (polar bear liver)

• Endocrine disorders: Obesity, hyperthyroidism, Cushing syndrome, hypoparathyroidism
• Thrombosis of the dural venous sinuses as a result of head trauma, otitis media, mastoiditis, or obstruction of jugular veins in the superior vena cava syndrome
41. What treatment is recommended for severe cases of idiopathic intracranial hypertension?
Patients with sustained visual field loss or severe refractory headache are candidates for treatment. Specific treatment depends on the presence of an identifiable precipitant, which should be removed when possible. For example, the cessation of the offending medication (e.g., tetracycline) or weight reduction in obese patients is recommended. Nonspecific treatment includes the administration of acetazolamide, furosemide, or hydrochlorothiazide and, sometimes, corticosteroids. In severe cases, surgical intervention is available through installation of a lumboperitoneal shunt or optic nerve sheath decompression.

Rogers DL: A review of pediatric idiopathic intracranial hypertension, Pediatr Clin North Am 61:579–590, 2014.

CLINICAL ISSUES
42. What are the key questions in a neurologic evaluation?
• Localization of the lesion (Where is the lesion?)
• Identification of the lesion (What is the lesion?)
• Time course of the disorder (Is it paroxysmal, acute, subacute, or chronic?)
• Presence of any regression (Is there a worsening of previously learned skills?)
• Development of the nervous system (Is it age appropriate?)
43. What are general rules that govern localization of a potential neurologic problem? Localization starts with the neurologic examination. The questions that need to be addressed are (1) is the examination normal or abnormal, and if abnormal, (2) is the abnormality focal, multifocal, or diffuse?
A problem can occur anywhere along the neuron axis: cerebrum, cerebellum, brainstem, spinal cord, nerve, neuromuscular junction, and muscle.
• Cerebrum: May present with seizures, mental status changes, headaches, unilateral signs (such as hemiparesis)
• Cerebellum: May involve ataxia, disturbances of speech, disorders of limb movement, nystagmus
• Brainstem: Combination of cranial nerve abnormalities and long-tract signs (symmetric weakness with or without sensory changes)
• Spinal cord: Defined level of impairment with motor and/or sensory changes below involved area and normal examination above
• Neuropathy: Distal more than proximal weakness with or without sensory changes
• Muscle disease: Proximal more than distal weakness with decreased deep tendon reflexes and normal sensation

Goldstein JL: Pediatric neurology in the emergency department: localization followed by differential diagnosis, Clin Pediatr Emerg Med 9:87, 2008.

44. What distinguishes the pediatric neurologic examination?
Observation. The most useful information is often acquired by watching the child MOVE and play. The level of interaction, creativity, and degree of sustained attention can be observed and are all important components of the mental status examination. By observing eye movements, response to sounds, the child’s reaction to visual stimuli introduced into the peripheral visual field, and the symmetry of facial movements, most of the cranial nerves can be tested. Persistent asymmetries of spontaneous motor activity (e.g., consistently reaching across midline for an object) are reliable signs of weakness. Inspection of the seated posture and gait of the child provides an assessment of the cerebellum and cerebellar outflow pathways.
45. What are the advantages and disadvantages of various imaging procedures used in pediatric neurologic evaluation?
• Skull films are useful for detection of fractures, lytic lesions, and widened sutures. They have poor sensitivity and specificity for intracranial pathology in the setting of trauma.

• CT scan without contrast is the best imaging technique for neurologic emergencies to screen a patient with significant head trauma for skull fractures, signs of herniation, or acute intracranial hemorrhage. It can also be used to screen for acute strokes and subarachnoid hemorrhages. Midline or ventricular shifts due to masses and cerebral edema or increased ICP can be noted. CT identifies bone clearly. This rapid study allows routine monitoring and is less expensive than MRI. There is a small but defined risk associated with radiation from CT scans.
• CT scan with contrast uses radiodense contrast material to allow better identification of disruptions in the blood-brain barrier or of highly vascular structures, significantly
improving detection of tumors, edema, focal inflammation, hemangiomas, and arteriovenous malformations.
• MRI without contrast is the preferred modality for most nonurgent examinations. It defines structures of brain more precisely than CT, especially within spinal cord, posterior fossa, and cisterns. It is more effective for subtle hemorrhages (especially subacute and chronic) and for tumors or masses. Different tissue-specific relaxation constants, called T1 and T2, and proton density allow for better definition of white and gray matter. It also provides an image in three dimensions. Its longer testing time may require sedation. Also, monitoring patients is more difficult in closed units. There are no known biologic hazards from MRI, which measures the emission of radio waves released when protons return to a lower energy state after excitation within characteristic tissue environments. MRI is contraindicated in patients with metallic implants that are ferromagnetic; this may heat and damage tissue.
• MRI with contrast is helpful in defining brain metastases and distinguishing postoperative scarring from other pathology.
• Magnetic resonance angiography (MRA) is a special type of MRI that displays larger arteries and veins without the use of contrast. It is less invasive than traditional arteriograms, and it is useful in defining arterial stenosis and identifying intracranial hemangiomas, arteriovenous malformations, and vascular aneurysms.
• MR spectroscopy (MRS) allows for in VIVO examination of some chemical constituents of the brain including N-acetylaspartate (NAA), a neuronal marker; choline; creatine; and lactate (a marker of energy metabolism).
• Functional MRI (fMRI) allows for in VIVO anatomic localization of the motor and sensory cortex, the visual cortex, and components of expressive and receptive language.
• Positron emission tomography (PET) detects localized functional abnormalities using short half-life isotopes of carbon, nitrogen, oxygen, and fluorine. Labeled glucose ligands are useful in the evaluation of epileptic foci before surgery. These are areas of reduced cerebral glucose metabolism during interictal periods. Evaluation of specific isotopes, such as the uptake value of alpha-[11C] methyl-L-tryptophan (AMT) for the epileptic tuber in tuberous sclerosis, may be beneficial in certain disorders.
• Single-photon emission computed tomography (SPECT) uses gamma-ray emission of lipophilic isotopes in the measurement of cerebral blood flow and is also used in the study of refractory epilepsies.
• Magnetoencephalography (MEG) provides real-time measures of neuronal electrical activity (localization of epileptic focus).

Menkes JH, Moser FG, Maria BL: Neurologic examination of the child and infant. In Menkes JH, Sarnat HB, Maria BL, editors: Child Neurology, ed 7, Philadelphia, 2006, Lippincott Williams & Wilkins, pp 18–27.

46. A child presents with progressive left leg weakness and diplopia, especially when looking toward the left. Where is the lesion?
The described history, in combination with an examination showing upper motor neuron nerve dysfunction, long tract signs, brisk reflexes, upgoing toe (Babinski sign), and a contralateral third-nerve palsy (down and out), localizes the lesion to the right pyramidal tract before the decussation (crossing over) and involves a lesion of the right third-nerve nucleus. The progressive course suggests a slow- growing lesion, such as a pontine glioma.
47. A dilated and unreactive pupil indicates the compression of what structure?
The third cranial nerve. This may be the result of compression anywhere along the course of the nerve. Uncal herniation is a medial displacement of the uncus of the temporal lobe and may cause this sign.

48. Pinpoint pupils and respiratory changes indicate the compression of what structure? Progressive central herniation of the brain downward through the foramen magnum causes compression of the pons and can produce this finding.
49. How does the presentation of stroke differ between infants and older children? Infants usually have a seizure, whereas older children have acute hemiplegia. Neonates almost always present with focal seizures.

Calder K, Kokorowski P, Tran T, Henderson S: Emergency department presentation of stroke, Pediatr Emerg Care 19:320– 328, 2003.
Children’s Hemiplegia and Stroke Association: www.chasa.org. Accessed on March 5, 2015. Pediatric Stroke Network: www.pediatricstrokenetwork.com. Accessed on March 11, 2015.

50. What is the differential diagnosis of stroke in children?
CEREBROVASCULAr disease, or stroke, can be the result of primary vascular disease, bleeding disorder (hemorrhagic stroke), or a variety of secondary problems that lead to thrombotic or embolic occlusions (most commonly of the middle cerebral artery). Diagnostic possibilities include the following:
• Cardioembolic: Cyanotic congenital heart disease, atrial myxoma, endocarditis, rheumatic or other valvular heart disease
• Hematologic: Hemoglobinopathies (especially sickle cell disease), hypercoagulable states (antithrombin III deficiency, protein C or S deficiency), hyperviscosity (leukemia, hyperproteinemia, thrombocytosis), coagulation disorders (lupus-associated antibodies, hemophilia, thrombocytopenia, factor V abnormalities, hyperhomocysteinemia)
• Circulatory: Vasculitis (infectious or inflammatory), occlusive (homocystinuria, arteriosclerosis, fibromuscular dysplasia of the internal carotid artery, posttraumatic carotid scarring), carotid or vertebral artery dissection, moyamoya disease, atrioventricular malformation with steal syndrome, anomalous circulation, posttraumatic air embolism, arterial aneurysm, hemiplegic migraine
• Metabolic: Mitochondrial disease

Gumer LB, Del Vecchio M, Aronoff S: Strokes in children: a systematic review, Pediatr Emerg Care 30:660–664, 2014. Freundlich CL, Cervantes-Arsianian AM, Dorfman DH: Pediatric stroke, Emerg Med Clin North Am 30:805–828, 2012.

51. What is the derivation of “moyamoya” in moyamoya disease?
Moyamoya, which is Japanese for “puff of smoke,” refers to the cerebral angiographic appearance of patients with this primary, vascular disease that results in stenosis of the arteries of the circle of Willis and carotid arteries, resulting in prominent arterial collateral circulation. It occurs in a wide variety of conditions, such as neurofibromatosis type 1, sickle cell disease, Down syndrome, and tuberous sclerosis, in addition to the idiopathic, likely genetic, vasculopathy that is endemic in Japan. Because it is a chronic condition, fine vascular collaterals can develop, and it is these collaterals that create the “puff of smoke” appearance on angiography (Fig. 13-2).

Figure 13-2. Injection of the internal carotid artery (ICA) demonstrates findings consistent with moyamoya disease: stenosis of the distal ICA (arrowhead), diminished filling of the middle and anterior cerebral artery branches, and the proliferation of collateral vessels, the “puff of smoke” finding. (From Winn HR, editor: Youmans Neurological Surgery, ed 6. Philadelphia, 2011, Saunders ELSEVIER, p 2145.)

Patients with moyamoya present with transient ischemic attacks, ischemic strokes, and seizures, although in children, the predominant manifestation is ischemia.

Kleinloog R, Regli L, Rinkel GJ, Klijn CJ: Regional differences in incidence and patient characteristics of moyamoya disease, a systematic review, J Neurol Neurosurg Psychiatry 83:531–536, 2012.
Scott RM, Smith ER: Moyamoya disease and moyamoya syndrome, N Engl J Med 360:1226–1237, 2009.

52. A child who develops weakness, incontinence, and ataxia 10 days after a bout of influenza likely has what diagnosis?
Acute disseminated encephalomyelitis (ADEM) is thought to be a postinfectious or parainfectious process that is targeted against central myelin. Any portion of the white matter may be affected. Multiple lesions with a perivenular lymphocytic and mononuclear cell infiltration and demyelination are seen on pathologic examination. ADEM has been associated with mumps, measles, rubella, varicella-zoster, influenza, parainfluenza, mononucleosis, and some immunizations. An associated transverse myelitis may be acute (developing over hours) or subacute (developing over 1 to 2 weeks), with both motor and sensory tract involvement. Bladder and bowel dysfunction are often early and severe. CSF examination shows a mild increase of pressure and up to 250 cells/mm3, with a predominance of lymphocytes. The MRI shows an increased T2 signal intensity. Prognosis, particularly with the use of intravenous corticosteroids, is generally good.

53. In patients with acute injury to the brain, what two types of edema may occur?
• Vasogenic edema results from increased permeability of the capillary endothelium with resulting exudation. It is more marked in cerebral white matter and occurs as a result of inflammation (meningitis and abscess), focal processes (hemorrhage, infarct, or tumor), vessel pathology, or lead or hypertensive encephalopathy. On cranial CT scan, vasogenic edema shows up best with the administration of contrast.
• Cytotoxic edema results from the rapid swelling of cells, especially astrocytes, and also from neurons and endothelial cells as a result of dysfunction of the membranes and ionic pumps from energy failure, which may lead to cellular death. Hypoxia caused by cardiac arrest, hypoxic-ischemic encephalopathy (HIE), various toxins, severe infections, status epilepticus, infarct, or increased ICP is also a possible cause.

54. What are the treatments for increased ICP?
• Hyperventilation: The usual goal is to lower the PCO2 to 25 to 30 mm. This causes vasoconstriction, which decreases the intracranial vascular volume.
• Fluid restriction, osmotic diuretics, and hypertonic mannitol solution all work to shrink brain water content, provided there is an intact blood-brain barrier (none of these are evidence-based for newborn infants).
• Head elevation in a midline position to 30 degrees maximizes venous return. Head elevation may worsen ICP in the presence of hypovolemia.
• External ventricular drain (EVD) is sometimes placed, both to monitor pressure and to allow for a minimal amount of CSF withdrawal. An EVD may typically be used with intraventricular hemorrhage.
• Normalization of physiologic parameters: It is important to avoid significant hypotension, hypoxia, hypoglycemia, and hyperthermia.

55. How is brain death defined?
Brain death is defined by an irreversible absence of cortical and midbrain activity. Determination of brain death in term newborns, infants, and children is a clinical diagnosis based on the absence of neurologic function with a known irreversible cause of coma. Spinal cord, peripheral nerve, or reflex muscular activity may persist despite brain death. Decorticate or decerebrate posturing, however,
is inconsistent with brain death. The examination must remain unchanged over time. Other countries have defined brain death as the absence of brainstem function alone, but in the United States, the absence of cortical function also must be demonstrated. The clinical hallmark of brain death is deep, unremitting, unresponsive coma.

De Georgia MA: History of brain death as death: 1968 to the present, J Crit Care 29:673–678, 2014. Banasiak KJ, Lister G: Brain death in children, Curr Opin Pediatr 15:288–293, 2003.

56. How is the diagnosis of brain death in children made? Patients with suspected brain death should be observed and tested on two separate occasions (including neurologic exam and apnea testing) over 12 to 24 hours for the following:
• Unresponsive coma and the absence of eye opening, extraocular movements, vocalizations, or other cerebral-generated activity
• The complete absence of brainstem function, including nonresponsive, midposition, or fully dilated pupils; no spontaneous or reflexive eye movements on oculovestibular testing (“doll’s eyes” and calorics); no bulbar muscle function (i.e., corneal, gag, cough, sucking, and rooting reflexes); and no respirations on apnea testing
• Apnea testing with a rise of arterial PCO2 at least 20 mm Hg above baseline and > 60 mm Hg overall with no respiratory effort
• Ancillary testing (EEG and radionuclide cerebral blood flow) is not required and should not be used as a substitute, but may be used to supplement the neurologic examination and apnea testing.

Nakagawa TA, Ashwal S, Mathur M, et al: Clinical report—Guidelines for the determination of brain death in infants and children: an update of the 1987 Task Force recommendations, Pediatrics 128:e720–e740, 2011.

57. Compare the persistent vegetative state with the minimally conscious state.
• The persistent vegetative state is “a form of eyes-open permanent unconsciousness in which the patient has periods of wakefulness and physiologic sleep/wake cycles, but at no time is the patient aware of himself or herself or the environment.” If this state persists for more than 3 months in children, the long-term outlook is grim.
• The minimally conscious state occurs on emergence from the persistent vegetative state, and a patient must demonstrate a reproducible action in one or more of four types of behavior: (1) simple command following; (2) gestural or verbal “yes/no” responses; (3) intelligible verbalization; or (4) purposeful behaviors.

Hirschberg R, Giacino JT: The vegetative and minimally conscious states: diagnosis, prognosis and treatment, Neurol Clin 29:773–786, 2011.

58. What is the differential diagnosis of an intracranial bruit?
An intracranial bruit can be found in normal children and may be augmented by contralateral carotid compression. Disorders that may be associated with an intracranial bruit include vascular malformations and conditions characterized by increased cerebral blood flow:
• Fever
• Cerebral angioma
• Intracerebral tumors
• Thyrotoxicosis
• Cerebral aneurysm
• Any cause of increased ICP
• Anemia
• Cerebral arteriovenous malformations
• Meningitis
• Cardiac murmurs

Mace JW, Peters ER, Mathies AW Jr: Cranial bruits in purulent meningitis in childhood, N Engl J Med 278: 1420–1422, 1968.

59. In a previously normal child who develops acute ataxia, what are the two most common diagnoses?
• Drug ingestion, especially of AEDs, heavy metals, alcohol, and antihistamines, is a common diagnosis.
• Acute postinfectious cerebellitis, most commonly after varicella, is a diagnosis of exclusion if drug screening, CT or MRI, CSF evaluation, and other tests are negative.

Prasad M, Ong MT, Setty G, et al: 15 minute consult: The child with acute ataxia, Arch Dis Child Educ Pract Ed 98: 212–216, 2013.

60. What are the causes of toe walking?
• CP (spastic diplegia)
• Hereditary spastic paraplegia (HSP)
• Muscular dystrophy
• Spinal dysraphism
• Hereditary or acquired polyneuropathies
• Intraspinal and filum terminale tumor
• Equinovarus deformity
• Isolated congenital shortening of the Achilles tendon
• Variation of normal in early stages of walking
• Normal development pattern in some toddlers

Oetgen ME, Peden S: Idiopathic toe walking, J Am Acad Orthop Surg 20:292–300, 2012.

61. What is the significance of a positive Babinski response?
Stimulation of the lateral aspect of the sole of the foot to the distal metatarsals may elicit an extensor plantar response (i.e., dorsiflexion of the big toe), known as a positive Babinski response or sign. Its presence can be normal up to 12 months of age. When it persists after this age, it can be a sign of disturbed pyramidal tract function. The stimulus elicits a number of sensory pathways with competing functions (including grip and withdrawal) and is somewhat dependent on the state of the infant and the examiner’s technique. Its value as a localizing sign in the neonate is more controversial, but a consistent asymmetry is abnormal.
62. A 7-year-old child with progressive ataxia, kyphoscoliosis, nystagmus, pes cavus (high arch), and an abnormal electrocardiogram (ECG) likely has what diagnosis? Friedreich ataxia. This heredodegenerative disease is an autosomal recessive disorder with childhood onset of gait ataxia, absent tendon reflexes, and extensor plantar responses. The spinal cord shows degeneration and sclerosis of the spinocerebellar tracts, the posterior column, and the corticospinal tracts. The condition is rare. The gene for Friedreich ataxia has been mapped to chromosome 9q13, contains a trinucleotide repeating sequence (GAA), and encodes for a protein called frataxin. A deficiency of frataxin leads to the accumulation of iron in the mitochondria and to oxidative stress, which leads to cell death.

Anheim M, Tranchant C, Koenig M: The autosomal recessive cerebellar ataxias, N Engl J Med 366:636–646, 2012. Alper G, Narayanan V: Friedreich’s ataxia, Pediatr Neurol 28:335–341, 2003.

63. What clinical features help distinguish peripheral from central vertigo?
Peripheral VERTIGO implies dysfunction of the labyrinth or vestibular nerve, whereas central VERTIGO is associated with abnormalities of the brainstem or temporal lobe.
Peripheral
• Hearing loss, tinnitus, and otalgia may be associated.
• Past pointing and falling in the direction of unilateral disease occur.
• In bilateral disease, ataxia occurs with the eyes closed.
• Vestibular and positional nystagmuses are present.
Central
• Cerebellar and cranial nerve dysfunction are frequently associated.
• No hearing loss is present.
• An alteration of consciousness may be associated.
• Vertigo is a common symptom with migraine.

Fenichel GM: Clinical Pediatric Neurology: A Signs and Symptoms Approach, ed 6. Philadelphia, 2009, Elsevier, p 365.

64. In what settings is hyperacusis noted? Hyperacusis, or increased sensitivity to sound, is found in patients with injury to the facial nerve (CN VII), which innervates the stapedius muscle, or in those with injury to the trigeminal nerve (CN V), which innervates the tensor tympani muscle. Exaggerated startle response to sound or vibration occurs in patients with lysosomal storage diseases (e.g., sphingolipidoses such as Tay-Sachs disease, GM1

gangliosidosis, and Sandhoff disease), Williams syndrome, hyperkalemia, tetanus, and strychnine poisoning.
65. What is the most common cause of neonatal asymmetric crying facies? In this entity with an incidence of approximately 1 per 160 live births, one side of the lower lip depresses during crying (on the normal side), and the other does not (Fig. 13-3). Often misdiagnosed as a facial nerve palsy resulting from forceps delivery, the most common cause is congenital absence or hypoplasia of the depressor anguli oris muscle of the lower lip. Although it is usually an isolated finding in most cases, the condition can be associated with 22q11.2 deletion syndrome and other congenital malformations, especially of the cardiovascular system.

Pasick C, McDonald-McGinn DM, Simbolon C, et al: Asymmetric crying facies in the 22q11.2 deletion syndrome: implications for future screening, Clin Pediatr 52:1144–1148, 2013.
Sapin SO, Miller AA, Bass HN: Neonatal asymmetric crying facies: a new look at an old problem, Clin Pediatr
44:109–119, 2005.

Figure 13-3. Asymmetric crying facies. (From Terzis JK, Anesti K: DEVELOPMENTAl facial paralysis: a REVIEW, J Plast Reconst Aesthet Surg 64:1318–1333, 2011, p 1324.)

66. What are the common causes of peripheral seventh-nerve palsy? Facial weakness caused by a lesion of the facial nerve (cranial nerve VII) is common. The facial weakness involves both the upper and lower face and affects both emotional and volitional facial movements. Any part of the nerve can be disturbed: the nucleus itself, the axon as it passes through the pons, or the peripheral portion of the nerve. A seventh-nerve palsy is either central or peripheral in location. Etiologies include the following:
• Trauma
• Developmental hypoplasia or aplasia, including the M€obius anomaly
• Bell palsy (usually idiopathic, but may follow nonspecific viral infections)
• Infections, including Ramsay Hunt syndrome (herpes zoster invasion of the geniculate ganglion producing herpetic vesicles behind the ear and painful paralysis of the facial nerve); Lyme disease; local invasion from suppurative mastoiditis or otitis media; mumps, varicella, Epstein-Barr virus, cytomegalovirus, rubella, human immunodeficiency virus, or enterovirus neuritis; sequelae of bacterial meningitis; and parotid gland infection, inflammation, or tumor
• Guillain-Barré syndrome
• Tumor of the brainstem or cerebellar pontine angle tumors
• Inflammatory disorders such as sarcoidosis

67. How is peripheral seventh-nerve palsy distinguished from central seventh- nerve palsy?
The patient with a suspected palsy is asked to wrinkle the forehead, raise the eyebrows, and close the eyes tightly. In peripheral seventh-nerve palsy, no forehead furrows are noted, and the affected eye does not open as wide as the unaffected eye. In central seventh-nerve palsy, forehead furrowing and relatively good eye opening occur because the cells of the facial nucleus that innervate the upper face receive bilateral innervation from fibers from both cerebral hemispheres. Lower facial muscles are innervated from only the single contralateral cerebral hemisphere.

Gilden DH: Bell’s palsy, N Engl J Med 351:1323–1331, 2004.

68. During recovery from Bell palsy, why do the eyes water at mealtime?
These are crocodile tears. The facial nerve supplies autonomic motor function to the lacrimal and salivary glands. Because of aberrant reinnervation during the course of healing from a facial nerve palsy, tasting a meal can trigger tearing rather than salivation. Folklore has it that crocodiles feel compassion for their victims and weep while munching. This is called synkinesis, which may also involve the other muscles of the facial nerve or may be congenital.
69. When are “doll’s eyes” movements considered normal or abnormal?
The OCULOVESTIBULAr reflex (also called oculocephalic, propriocepTIVE head-turning reflex, or doll’s eyes reflex) is used most commonly as a test of brainstem function. The patient’s eyelids are held open while the head is briskly rotated from side to side. A positive response is contraversive conjugate eye deviation (i.e., as the head rotates to the right, both eyes deviate to the left). Doll’s eyes movements are interpreted as follows:
• In healthy awake newborn infants (who cannot inhibit or override the reflex with willful eye movements), the reflex is easy to elicit and is a normal finding. It can be used to test the range of the extraocular movements of infants during the first weeks of life.
• In healthy, awake, mature individuals, normal vision overrides the reflex, which is thus normally absent, and so the eyes follow the head turning.
• In a patient in a coma with preserved brainstem function, the depressed cortex does not override the reflex, and doll’s eyes movements occur in rapid head rotation. Indeed, the purpose of eliciting this reflex in the comatose patient is to demonstrate that the brainstem still functions normally.
• In a patient in a coma with brainstem damage, the neural circuits that carry out the reflex are impaired, and the reflex is abolished.
70. How are cold calorics done?
As a test of brainstem function in an obtunded or comatose individual, 5 mL of ice-cold water is placed in the external ear canal (after ensuring the integrity of the tympanic membrane), with the head elevated at 30 degrees. A normal response occurs with deviation of the eyes to the side in which the water was placed. No response indicates severe dysfunction of the brainstem and the medial longitudinal fasciculus. If done in the waking state, ipsilateral deviation occurs with nystagmus in the opposite direction.
71. What causes pinpoint pupils?
Pupillary size represents a dynamic balance between the constricting influence of the third nerve (representing the parasympathetic autonomic nervous system) and the dilating influence of the ciliary nerve (which conducts fibers of the sympathetic nervous system). Pinpoint pupils indicate that the constricting influence of the third cranial nerve is not balanced by opposing sympathetic dilation. Possible etiologies include the following:
• Structural lesion in the pons through which the sympathetic pathways descend (most commonly, hemorrhage)
• Opiates, such as heroin or morphine
• Other agents, including propoxyphene, organophosphates, carbamate insecticides, barbiturates, clonidine, meprobamate, pilocarpine eyedrops, and mushroom or nutmeg poisoning
72. What is the differential diagnosis of ptosis?
Ptosis is the downward displacement of the upper eyelid as a result of dysfunction of the muscles that elevate the eyelid. A drooping eyelid may represent pseudoptosis caused by swelling of the eyelid as a

result of local edema or active blepharospasm. True ptosis results from weakness of the eyelid muscles or interruption of its nerve supply. Etiologies include the following:
• Muscular: Congenital ptosis, which may occur alone or in the setting of Turner or Smith-Lemli-Opitz syndrome, myasthenia gravis (associated with marked daytime fluctuation), botulism, or some muscular dystrophies
• Neurologic: Horner syndrome, which results from the interruption of the sympathetic supply to M€uller smooth eyelid muscle, and third-nerve palsy, which innervates the levator palpebral muscle; brainstem or orbital tumor (concerning if blurred vision also present)
73. What does the Marcus Gunn pupil detect?
An afferent pupillary defect (APD). The pupils are normally equal in size (except for patients with physiologic anisocoria) as a result of the consensual light reflex: light entering either eye produces the same-strength “signal” for the constriction of both the stimulated and nonstimulated pupil. Some diseases of the maculae or optic nerves affect one side more than the other, such as multiple sclerosis or optic neuritis. For example, a meningioma may develop on one optic nerve sheath. As a result of unilateral or asymmetric optic nerve dysfunction, a Marcus Gunn pupil may result.

74. How is the swinging flashlight test done to detect a Marcus Gunn pupil?
• The patient is examined in a dim room, and fixation is directed to a distant target. This permits maximal pupillary dilation because of a lack of direct light and accommodation reflexes.
• Light presented to the “good” eye produces the equal constriction of both pupils. A flashlight is swung briskly over the bridge of the nose to the eye with the “defective” optic nerve. The abnormal pupil remains momentarily constricted from the lingering effects of the consensual light response. However, the impaired eye with its reduced pupillomotor signal soon escapes the consensual reflex and actually dilates, despite being directly stimulated with light. The pupil that paradoxically dilates to direct light stimulation displays the afferent defect.

EPILEPSY
75. What is epilepsy? Epilepsy describes a syndrome of recurrent, unprovoked seizures, typically two or more, not the result of fever or a systemic medical condition. It is derived from the Greek verb epilepsia meaning “to seize upon” or “to take hold of.” The early Greeks referred to it as the sacred disease, but Hippocrates debunked this notion and argued from clinical evidence that it arose from the brain. Epilepsy is not an entity or even a syndrome but rather a symptom complex arising from disordered brain function that itself may be the result of a variety of pathologic processes.

Chang BS, Lowenstein DH: Epilepsy, N Engl J Med 349:1257–1266, 2003. American Epilepsy Society: www.aesnet.org. Accessed on March 14, 2015. Epilepsy Foundation: www.epilepsy.com. Accessed on March 5, 2015.

76. What is the long-term outcome for children with epilepsy?
There are many different causes of epilepsy, and, in large part, the outcome relates to the underlying etiology. Children with idiopathic or genetically determined epilepsy have the best prognosis, whereas children with antecedent neurologic abnormalities fare less well. Nearly 75% of children will enter into a sustained remission 3 to 5 years after the onset of their epilepsy. There is no evidence that antiepileptic medications as they are currently used in clinical practice are neuroprotective or that they alter the long- term outcome of patients. Although there is a favorable prognosis for the remission of seizures, children with epilepsy are at an increased risk for having other long-term comorbidities, including difficulties achieving social, educational, and vocational goals. Treatment with antiepileptic medications is one important part of the management of the child, but other critical aspects of the physician–patient interaction, including educating, counseling, and advocacy, are equally important.

77. How often are EEGs abnormal in healthy children? About 10% of “normal” children have mild, nonspecific abnormalities in background activity. About 2% to 3% of healthy children have unexpected incidental epileptiform (i.e., spikes or sharp wave) patterns. Some may have heritable, familial EEG abnormalities without a clinical seizure disorder (e.g., centrotemporal spikes seen in benign seizure-susceptibility syndromes such as rolandic epilepsy). In patients with migraines, the EEG may frequently have epileptiform features.

78. Should an EEG be done on all children who have a first afebrile seizure? This is a major controversial issue. Of new-onset seizures in children, about one-third do not involve fever. The American Academy of Neurology has recommended that all children with a first seizure without fever undergo an EEG in an effort to better classify the epilepsy syndrome. Others argue that the quantity of expected information from obtaining EEGs for all cases is too low to affect treatment recommendations in most patients. They suggest that a selective approach to EEG use should be pursued, particularly for children with a seizure of focal onset, for children younger than 1 year, and for any child with unexplained cognitive or motor dysfunction or abnormalities on neurologic examination.

Khan A, Baheerathan A: Electroencephalogram after first unprovoked seizure in children: Routine, unnecessary or case specific, J Pediatr Neurosci 8:1–4, 2013.
Hirtz D, Ashwal S, Berg A, et al: Practice parameter: evaluating a first nonfebrile seizure in children. Report of the Quality Standards Subcommittee of the American Academy of Neurology, the Child Neurology Society, and the American Epilepsy Society, Neurology 55:616–623, 2000.

79. In a patient with a suspected seizure disorder, but a normal EEG, how can the sensitivity of the EEG be increased?
• Repeat the EEG.
• Obtain following sleep deprivation.
• Use hyperventilation and photic stimulation (e.g., strobe lights).
• Obtain a continuous video EEG (>24 hours).
80. Which types of epilepsy are characterized by specific EEG findings?
• Rolandic epilepsy: Bicentral spikes (only during sleep in 30%); “midtemporal spikes” is a misnomer, related to the positioning of the EEG electrodes in the temporal regions.
• Benign epilepsy with occipital focus: Continuous unilateral or bilateral occipital high-voltage spike waves
• Panayiotopoulos syndrome: Also known as early-onset occipital epilepsy; abnormal spikes in one or both occipital lobes.
• Absence epilepsy: Characteristic 3-Hz spike-wave pattern
• Juvenile absence epilepsy: Characteristic fast spike-wave pattern (>3 Hz)
• Juvenile myoclonic epilepsy: Fast spikes and polyspike-wave patterns
• Infantile spasms: Hypsarrhythmia, a markedly disorganized pattern
• Lennox-Gastaut syndrome: Slow spike-wave forms at <3-Hz frequency
• Landau-Kleffner syndrome: Sleep activated spike-wave discharges, may be in pattern of electrical
status epilepticus of sleep (ESES)
81. Should all children with a new-onset afebrile, unprovoked generalized seizure have a CT or MRI evaluation?
Although most adults with new-onset seizures should have a head imaging study (preferably MRI), the relatively high frequency of idiopathic seizure disorders in children often obviates a scan in those with generalized seizures, nonfocal EEGs, and normal neurologic examinations. Consider obtaining a cranial imaging study in the following situations:
• Any seizure with focal components (other than mere eye deviation)
• Newborns and young infants with seizures
• Status epilepticus at any age
• Focal slowing or focal paroxysmal activity on EEG

Weeke LC, Groenendaal F, Toet MC, et al: The aetiology of neonatal seizures and the diagnostic contribution of cerebral magnetic resonance imaging, DEV Med Child Neurol 57:248–256, 2015.
Hirtz D, Berg A, Bettis D, et al: Quality Standards Subcommittee of the American Academy of Neurology; Practice Committee of the Child Neurology Society: Practice parameter: treatment of the child with a first unprovoked seizure. Report of the Quality Standards Subcommittee of the American Academy of Neurology and the Practice Committee of the Child Neurology Society, Neurology 60:166–175, 2003.

82. Which disorders commonly mimic epilepsy? Many conditions are characterized by the sudden onset of transient abnormal consciousness, awareness, reactivity, behavior, posture, tone, sensation, or autonomic function. Syncope, breath-holding spells, migraine, hypoglycemia, narcolepsy, cataplexy, sleep apnea, gastroesophageal reflux, and parasomnias (night terrors, sleep walking, sleep talking, nocturnal enuresis) feature an abrupt or “paroxysmal” alteration of brain function and suggest the possibility of epilepsy.

83. What are ways to distinguish psychogenic nonepileptic seizures (PNES) from epileptic seizures?
PNES consist of changes in behavior (including motor manifestations) or consciousness that are not accompanied by electrophysiological changes. PNES were formerly called pseudoseizures or hysterical seizures, but these terms are now discouraged. Features that help identify PNES include:
History: Patient is more likely to have a history of psychiatric problems including depression, anxiety, posttraumatic stress disorder and somatoform disorder. Common precipitating factors: school-related difficulties and interpersonal conflict.
Clinical: PNES are usually longer than 2 minutes, eyes are forcefully closed (compared to eyes typically open in epileptic seizures), motor activity is waxing and waning, vocalizations are commonly present (uncommon in epileptic seizures), incontinence is less common, awakening and reorientation are more rapid than epileptic seizures, and recall of event is more common in PNES than with an epileptic seizure.
Studies: Video-EEG (most reliable to rule-out epileptic seizures); ambulatory EEG; prolactin levels (typically elevated 15-20 minutes after an epileptic seizure but not after PNES).

Reilly C, Menlove L, Fenton V, Das KB: Pyschogenic nonepileptic seizures in children: a review, Epilepsia 54:1715–1724, 2013. Avbersek A, Sisodiya S: Does the primary literature provide support for clinical signs used to distinguish psychogenic nonepileptic seizures from epileptic seizures? J Neurol Neurosurg Psychiatry 81:719–725, 2010.
Korff CM, Nordlii DR Jr: Paroxysmal events in infants: persistent eye closure makes seizures unlikely, Pediatrics 116: e485–e486, 2005.
84. What are the categories of seizures in children?
The syndrome classification as codified in the International Classification of Epileptic Seizures by the International League Against Epilepsy (ILAE) distinguishes seizures on the basis of type rather than etiology (Table 13-2). The classification was revised in 2010 with an elimination of terms previously used for focal seizures such as “simple partial,” “complex partial,” and “partial seizures secondarily generalized” as being too imprecise. The current classification includes generalized seizures, which occur in and rapidly engage bilaterally distributed networks while focal seizures are more limited to one hemisphere, either discretely localized or more widely distributed in that hemisphere. While the classification for generalized seizures was straightforward, the ILAE believed that there was inadequate information to create a scientific classification for focal seizures, but that focal seizures should be described according to their manifestations (e.g., dyscognitive with impairment of consciousness/ awareness, focal motor).

Table 13-2. International League Against Epilepsy Classification of Seizures
Generalized
Tonic-clonic (in any combination) Absence
Typical Atypical
Absence with special features Myoclonic absence
Eyelid myoclonic Myoclonic Myoclonic Myoclonic atonic Myoclonic tonic Clonic
Tonic Atonic
Focal seizures
Unknown
Epileptic spasms (infantile spasms)
Adopted from Berg AT, BERKOVic SF, Brodie MJ, et al: REVISED terminology and concepts for organization of seizures and epilepsies: Report of the ILAE Commission on Classification and Terminology, 2005-2009, Epilepsia 51:678, 2010.

Additionally, previous terminology by the ILAE categorized the causes of seizures into symptomatic (due to a known disorder of the CNS), cryptogenic (due to a hidden or occult cause) or idiopathic (no known cause except a possible hereditary predisposition). This terminology was believed to be confusing and was eliminated. New etiologic categories involve three groups: genetic, structural/metabolic, and unknown.

Berg AT, Berkovic SF, Brodie MJ, et al: Revised terminology and concepts for organization of seizures and epilepsies: Report of the ILAE Commission on Classification and Terminology, 2005-2009, Epilepsia 51:676–685, 2010.

85. What are structural and metabolic causes of seizures?
See Table 13-3.

Table 13-3. Structural and Metabolic Causes of Seizures
Fever
• Simple febrile seizures
• Complicated febrile seizures
Trauma
• Impact seizures
• Early posttraumatic seizures
• Late posttraumatic seizures
Hypoxia
• Complicated breath-holding spells
• Hypoxic seizures
Metabolic
• Acquired metabolic disorders
• Neurologic effects of systemic disease
• Inborn errors of metabolism
Toxins
• Drugs
• Drug withdrawal
• Biologic toxins
Stroke
• Ischemic stroke
• Embolic stroke
• Hemorrhagic stroke
Intracranial Hemorrhage
• Subdural hemorrhage
• Subarachnoid hemorrhage
• Intracerebral hemorrhage
CNS Malformation
• Cerebral atrophy
• Cortical dysplasia
• Microcephaly
CNS Central nervous system.
Adapted from EVANS OB: Symptomatic seizures, Pediatr Ann 28:231–237, 1999.

86. If a previously normal child has an afebrile, generalized tonic-clonic seizure, what should parents be told about the risk for recurrence? Studies indicate that the recurrence rate is between 25% and 50%. The EEG is an important predictor of recurrence. A subsequent normal EEG reduces the 5-year recurrence risk to 25%. Occurrence of the seizure during sleep increases the risk to 50%. Half of recurrences will occur during the first 6 months after the first seizure; two thirds will occurs within 1 year, and 90% or more will have occurred within

2 years. The child’s age at the time of the first seizure and the duration of the seizure do not affect the recurrence risk.

Shinnar S, Berg AT, Moshe SL, et al: The risk of seizure recurrence after a first unprovoked afebrile seizure in childhood: an extended follow-up, Pediatrics 98:216–225, 1996.

87. What are the most common inherited seizure or epilepsy syndromes?
• Febrile convulsions
• Rolandic epilepsy, childhood absence epilepsy
• Juvenile myoclonic epilepsy (of Janz)

88. What are the clinical features of rolandic epilepsy? Rolandic epilepsy is an idiopathic localization-related epilepsy that represents 10% to 15% of all childhood seizure disorders.
• It begins in school-age children (4 to 13 years old) who are otherwise healthy and neurologically normal.
• Seizures are idiopathic or familial (autosomal dominant inheritance with age-dependent penetrance).
• Seizures may be simple or complex and partial or generalized. Classically, there is a history of one- sided facial paresthesias and twitching and drooling that may be followed by hemiclonic movements or hemitonic posturing. Consciousness is typically preserved. The seizures are primarily nocturnal and may secondarily generalize.
• Often referred to as benign because the individual is developmentally normal, seizures are usually rare and nocturnal, and they most often resolve after puberty.
• Many children may have one or only several seizures, so AED treatment is not mandatory, especially when seizures are nocturnal.
• Treatment is indicated when seizures occur in the daytime (diurnal) or if escalating in frequency.

89. What distinguishes typical and atypical absence seizures?
Typical absence
• EEG: 3-Hz spike wave; may be activated by hyperventilation or intermittent photic stimulation
• OBSERVATIONS: Abrupt onset and ending (typically 5 to 10 seconds)
• Simple subtype: Unresponsiveness with no other associated features except minor movements (e.g., lip smacking or eyelid twitching)
• Complex subtype: Unresponsiveness with more prolonged (>5 to 10 seconds) mild atonic, myoclonic, or tonic features or automatisms
Atypical absence (most common in Lennox-Gastaut syndrome)
• EEG: 2-Hz (or slower) spike wave
• OBSERVATIONS: Gradual onset and ending; frequency is more cyclic; unresponsive with more prolonged and pronounced atonic, tonic, myoclonic, or tonic activity

90. In a child who is suspected of having absence seizures, how can a seizure be elicited during an examination?
Hyperventilation for at least 3 minutes is a useful provocative maneuver to precipitate an absence seizure. Young patients may be coaxed into overbreathing by making a game of it. Hold a tissue paper or pinwheel in front of the child’s mouth, and then instruct the patient to keep breathing fast enough to keep the tissue aloft or the pinwheel spinning.

91. What percentage of patients with absence seizures also have occasional grand mal seizures?
About 30% to 50%; these may occur years later.

92. What is the prognosis for children with absence epilepsy?
The prognosis for patients with childhood absence epilepsy has been studied prospectively, and nearly 90% of patients who have normal intelligence, normal neurologic examination, normal EEG background activity, no family history of convulsive epilepsy, and no history of tonic-clonic convulsions will have seizures that remit. Conversely, the complete absence of favorable factors is associated with a poor

prognosis for the cessation of seizures. It may be that absence seizures are expressed on a spectrum from typical childhood absence epilepsy (with the typical absence seizure that is genetic in origin to the Lennox- Gastaut syndrome) to the atypical absence seizure, which is symptomatic of brain injury.
93. A teenager, like his father, develops brief, bilateral, intermittent jerking of his arms. What seizure disorder is he likely to have?
Juvenile myoclonic epilepsy, which is also called myoclonic epilepsy of Janz, is a familial form of primary idiopathic generalized epilepsy that typically involves “fast” 3- to 5-Hz spike-wave discharges on EEG (“impulsive petit mal”) and autosomal dominant inheritance. The distinctive clinical features of this type of epilepsy include morning myoclonic jerks, generalized tonic-clonic seizures upon awakening, normal intelligence, a family history of similar seizures, and onset between the ages of 8 and 20 years.
94. What are myoclonic seizures?
These seizures are characterized by rapid, bilateral, symmetric muscle contractions of short duration— “quick jerks.” They may be isolated, or they may occur repetitively. Myoclonic seizures may be the sole manifestation of epilepsy, or more commonly, they may be associated with absence seizures or tonic- clonic seizures. By definition, a myoclonic seizure lasts less than 100 μsec.

KEY POINTS: EPILEPSY
1. Definition: Repeated, unprovoked seizures
2. Classified as localization-related: focal or generalized
3. Main etiologic categories: genetic, structural/metabolic, and unknown
4. Compared with older children with new-onset seizures, infants are more likely to have results from electroencephalogram (EEG) and neuroimaging that affect diagnosis and prognosis.
5. Proper classification of epilepsy syndromes provides guidance for treatment options and prognoses.

95. What distinguishes atonic and akinetic seizures?
An atonic seizure involves the sudden and usually complete loss of tone in the limb, neck, and trunk muscles. Muscle control is lost without warning, and the child may be seriously injured. This situation is often aggravated by the occurrence of one or more myoclonic jerks immediately before muscle tone is lost so that the fall is associated with an element of propulsion. Atonic seizures are particularly common in children with static encephalopathies, and they may prove refractory to therapy. In an akinetic seizure, movement is arrested without a significant loss of muscle tone; this is rare.
96. Which seizure types constitute the “epileptic encephalopathies”? Epileptic encephalopathies constitute a group of diverse disorders that occur early in life with generalized or focal seizures resistant to pharmacology, persistent severe EEG abnormalities, and cognitive dysfunction with deterioration. The prototypical genetic epilepsy in this category is Dravet syndrome, also known as severe myoclonic epilepsy of infancy (SMEI), which is characterized by mutations in the sodium channel SCN1A gene. Other ion channel and non–ion-channel genetic defects are being identified. Other epileptic encephalopathies include Ohtahara syndrome, Lennox-Gastaut syndrome, Landau-Kleffner syndrome, and epileptic (infantile) spasms.

Nieh SE, Sherr EH: Epileptic encephalopathies: new genes and new pathways, Neurotherapeutics 11:796–806, 2014. Covanis A: Epileptic encephalopathies (including severe epilepsy syndromes), Epilepsia 53(Suppl 4): 114–126, 2012.

97. What is the classic triad of epileptic (infantile) spasms? Spasms, hypsarrhythmia, and developmental regression. Epileptic (infantile) spasms are also known as West syndrome. The condition is named for the physician who first described the condition in his own son in 1841.
98. What characterizes hypsarrhythmia? The term means “mountainous slowing,” and it describes the classic interictal EEG of epileptic (infantile) spasms that is characterized by extremely high-voltage (>300 μV), slow, and disorganized brain waves with multifocal spike activity. Hypsarrhythmia may either precede or follow the onset of epileptic spasms. This EEG configuration may appear first or most obviously in non–rapid eye movement sleep and confirms
the clinical diagnosis of epileptic spasms.
It should be noted that the presence of hypsarrhythmia is not a prerequisite for epileptic spasms.

99. How commonly is a cause identified in epileptic spasms?
A cause can be identified in up to 90% of children with epileptic spasms, particularly in those who are symptomatic at the time of the initial seizure. Of identifiable causes, three-fourths are prenatal or perinatal, and one-fourth are postnatal. All patients with epileptic spasms should have detailed neuroimaging and metabolic and genetic studies. Causes, including some possible specific examples, include the following:
• Prenatal and perinatal: Neurocutaneous disorders (tuberous sclerosis), brain injury (HIE), intrauterine infection (cytomegalovirus), brain malformations (lissencephaly, agenesis of the corpus callosum), inborn metabolic errors (nonketotic hyperglycinemia, phenylketonuria, maple syrup urine disease, pyridoxine dependency)
• Postnatal: Infectious (herpes encephalitis), HIE, head trauma

100. What is the prognosis for infants with epileptic spasms?
Prognosis in large part depends on the underlying etiology and the clinical state at the time of the first seizure. In the group with an unknown cause (10% to 15%), development, neurologic examination, and imaging studies are usually normal at the onset. With adrenocorticotropic hormone (ACTH) treatment, up to 40% will have a complete or near-complete recovery with normal cognitive development. In the group with a known structural/metabolic cause (85% to 90%), neurologic deficits, developmental delays, or cranial abnormalities are typically present before the first seizure. In this group, complete or near- complete recovery is achieved by only 5% to 15%. Twenty-five to 50% will develop Lennox-Gastaut syndrome.
Regression may be seen before the onset of epileptic spasms, especially in visual or motor function.

Widjaja E, Go C, McCoy B, Snead OC: Neurodevelopmental outcome of infantile spasms: a systematic review and meta- analysis, Epilepsy Res 109:155–162, 2015.
Kivity S, Lerman P, Ariel R, et al: Long-term cognitive outcomes of a cohort of children with cryptogenic infantile spasms treated with high-dose adrenocorticotropic hormone, Epilepsia 45:255–262, 2004.

101. What is the treatment of choice for epileptic spasms?
Currently in the United States, most children with epileptic spasms are treated with ACTH as the first treatment option with positive response rate of approximately three-fourths of patients. Vigabatrin has been shown to be superior to ACTH for children with tuberous sclerosis with a 95% spasm cessation rate. Vigabatrin may cause visual field defects, including visual field constriction and retinal toxicity, which may increase with duration of treatment and mandates periodic assessment with electroretinograms. Vigabatrin may also cause abnormal enhancement or restricted diffusion on MRI of the deep gray matter (thalamus, basal ganglia, and brainstem), which is reversible after cessation of treatment.

Hsieh DT, Jennesson MM, Thiele EA: Epileptic spasms in tuberous sclerosis complex, Epilepsy Res 106:200–210, 2013. Go CY, Mackay MT, Weiss SK, et al: Evidence-based guideline update: medical treatment of infantile spasms. Report of the Guideline Development Subcommittee of the American Academy of Neurology and the Practice Committee of the Child Neurology Society, Neurology 78:1974–1980, 2012.

102. What is the most likely diagnosis in a child of Ashkenazi descent with stimulus- sensitive seizures, cognitive deterioration, and a cherry-red spot?
The classic lysosomal lipid storage disorder presenting symptoms of a progressive encephalopathy during infancy is Tay-Sachs disease. The infantile forms of GM2 gangliosidosis includes Tay-Sachs disease, which is caused by a deficiency of hexosaminidase A, and Sandhoff disease, which is caused by a deficiency of hexosaminidase A and B. Tay-Sachs is an autosomal recessive disorder that is localized to chromosome 15, with an incidence of 1 in 3900 in the Ashkenazi Jewish population of Eastern or Central European descent. The enzymatic defect leads to intraneuronal accumulation
of GM2 ganglioside. Normal development is seen until 4 to 6 months of age, when hypotonia and a loss of motor skills occur, with the subsequent development of spasticity, blindness, and macrocephaly. The classic cherry-red spot is present in the ocular fundi of more than 90% of patients (Fig. 13-4).

Figure 13-4. A cherry red spot in a patient with GM1 gangliosidosis. Note the whitish ring of sphingolipid- laden ganglion cells surrounding the fovea. (From Kleigman RM, Stanton BF, Schor NF, et al, editors: Nelson Textbook of Pediatrics, ed 19. Philadelphia, 2011, ELSEVIER Saunders, p 2072.)

103. A patient with seizures, microcephaly, and a low CSF glucose but a normal serum glucose has what likely condition?
The GLUT-1 deficiency syndrome, previously referred to as the glucose transporter protein deficiency syndrome, was first described in 1991. The clinical phenotype is variable, but the child usually presents symptoms during the first years of life with seizures and delays of motor and mental development. Movement disorders, including dystonia, ataxia, myoclonus, and spasticity, are also seen. The head circumference decelerates during the first years of life. The diagnosis should be suspected if CSF reveals low glucose (and lactate) concentrations without evidence of inflammation, and serum blood sugars are normal. The ketogenic diet is the gold standard of treatment, although a modified Atkins diet has also been shown to be effective.

De Giorgis V, Vegglotti P: GLUT1 deficiency syndrome 2013: current state of the art, Seizure 10:803–811, 2013. Pong AW, Geary BR, Engelstad KM, et al: Glucose transporter type 1 deficiency syndrome: epilepsy phenotypes and outcomes, Epilepsia 53: 1503–1510, 2012.

104. What is the clinical triad of the Lennox-Gastaut syndrome?
Lennox-Gastaut syndrome is characterized by mental retardation, seizures of various types, and disorganized slow spike-wave activity on an EEG. The seizures usually begin during the first 3 years of life and are characteristically severe and refractory to anticonvulsant drugs. Prognosis is poor, with more than 80% of children continuing to have seizures into adulthood.

Arzimanoglou A, French J, Blume WT, et al: Lennox-Gastaut syndrome: a consensus approach on diagnosis, assessment, management, and trial methodology, Lancet Neurol 8:82–93, 2009.

105. A 5-year-old with a history of normal language development who develops seizures and inattention to speech with severe regression of language skills has what likely condition?
Landau-Kleffner syndrome. First described in 1957, this is a condition of acquired epileptic aphasia with nocturnal EEG abnormalities, reduction in language function, and problems with attention. Despite the use of various AEDs and/or ACTH, recovery is often delayed, and communication problems persistent.

Caraballo RH, Cejas N, Chamorro N, et al: Landau-Kleffner syndrome: a study of 29 patients, Seizure 23:98–104, 2014.

106. How is status epilepticus defined?
• Because of uncertainty regarding at precisely what time morbidity ensues in the course of a prolonged seizure, there is a VARIANCE in definitional length regarding status epilepticus. In general,
>30 minutes of continuous or sequential seizure activity has previously defined status epilepticus. The operational definition for status epilepticus, however, is 5 minutes, at which time a prolonged seizure is likely to become continuous, and thus treatment should be considered or started.
• Recurrent seizures without full recovery of consciousness between seizures.
107. Why is status epilepticus so dangerous? With the onset of a seizure, catecholamine release and sympathetic discharge result in increased heart rate and blood pressure. Cerebral flow increases dramatically to compensate for the increased metabolic needs of the brain. With persistence of the seizure, compensatory mechanisms begin to fail. Respiratory acidosis and metabolic acidosis develop. Systemic blood pressure falls. ICP increases. The inability to meet the increased oxygen demands of the brain results in an intracranial switch to anaerobic metabolism with acidosis, increased CSF lactate, and cerebral edema. The prolonged electrical discharges by themselves may also cause neuronal damage, referred to as excitotoxicity.

Miskin C, Hasbani DM: Status epilepticus: immunologic and inflammatory mechanisms, Semin Pediatr Neurol 21: 221–225, 2014.

108. What should be done in the first 10 minutes for a child who presents with an ongoing seizure?
• 0 to 5 minutes: Confirm the diagnosis. Maintain noninvasive airway protection by head positioning or oropharyngeal airway. Administer 100% nasal oxygen. Suction as needed. Obtain and frequently monitor vital signs using pulse oximetry and ECG. Establish an intravenous or intraosseous line. Obtain venous blood for laboratory determinations (e.g., glucose, serum chemistries, hematology studies, liver function studies, toxicology screen, culture, anticonvulsant levels if patient is a known epileptic). Administer antipyretics as indicated.
• 5 to 10 minutes: If hypoglycemic (or if a rapid reagent strip for glucose testing is not available), administer 2 mL/kg of D25W or 5 mL/kg of D10W. If IV/IO access is present, administer lorazepam, 0.1 mg/kg (max: 4 mg) IV/IO at 2 mg/min. If IV/IO access cannot be established, options include (1) diazepam: 2 to 5 years, 0.5 mg/kg; 6 to 11 years, 0.3 mg/kg;
> 12 years, 0.2 mg/kg (max: 20 mg); (2) midazolam: intranasal, 0.2 mg/kg (max: 10 mg); buccal,
0.5 mg/kg (max: 10 mg). Repeat lorazepam or one-half midazolam dose in 5 to 10 minutes if
seizure persists.

Abend NS, Loddenkemper T: Pediatric status epilepticus management, Curr Opin Pediatr 26:668–674, 2014.

109. What is the most common cause of refractory seizures?
An inadequate serum concentration of antiepileptic medication is the most common cause of persistent seizures, but other causes should be considered:
• Drug toxicity, especially with phenytoin, may manifest by deteriorating seizure control.
• Electrolyte disorders, especially with an acute illness, may be causative.
• Metabolic abnormalities, particularly in patients with inborn errors of metabolism, such as a mitochondrial disorder, should be considered.
• Medications may have a paradoxic reaction and exacerbate certain types of seizures, particularly in children with mixed seizure disorders. For example, carbamazepine or phenytoin may control generalized tonic-clonic seizures in patients with juvenile myoclonic epilepsy, but may aggravate myoclonic and absence seizures.
• Incorrect identification of the epilepsy syndrome may be a cause. Partial seizures may masquerade as a generalized form of epilepsy in the very young child (bilateral symmetric tonic posturing may be seen in partial seizures). Conversely, generalized forms of epilepsy may first appear as partial seizures (severe infantile myoclonic epilepsy). Treatment based on an epilepsy syndrome rather than ictal semiology usually improves control in these circumstances.
110. What is the role of the ketogenic diet for the treatment of seizures?
The ketogenic diet is effective for the treatment of all seizure types, particularly in children with myoclonic forms of epilepsy. The diet involves supplying most calories through fats, with concurrent

limitation of carbohydrates and protein. The mechanism of seizure control is unclear, but it is perhaps related to a switch in the cerebral metabolism from the use of glucose to the use of β-hydroxybutyrate. After 24 hours of fasting, the child is placed on a high-fat diet in which the ratio of fats to carbohydrates and protein combined is 3:1 to 4:1. Anticonvulsant drugs may be reduced or eliminated entirely if the diet is effective. The regimen must be followed closely, and parents must understand the demands of close adherence to the diet. A skilled dietitian is instrumental for providing variety and palatability to the diet. It is important to recall that the diet may have adverse effects, including serious, potentially life- threatening complications such as hypoproteinemia, lipemia, and hemolytic anemia. Variants of the ketogenic diet include the modified Atkins diet and the low glycemic index. β-Hydroxybutyrate levels are used to assess the degree of acidosis (similar to a drug level).

Kossoff EH, Zupec-Kania BA, Rho JM: Ketogenic diets: an update for child neurologists, J Child Neuro 24:979–988, 2009.

111. What is the role of the vagus nerve stimulator in seizure control?
The VAGUS NERVE stimulator (VNS) is a surgically implanted device that intermittently stimulates the left vagus nerve. Why this decreases seizure frequency is not well understood, although it causes alterations in epinephrine release and is thought to increase GABA levels in the brainstem. It is a palliative—not curative—procedure that has been performed in adults and children with intractable complex partial seizures or generalized tonic seizures that are not considered candidates for definitive surgical cure. The VNS has been placed in children as young as 2 to 3 years old.

Elijamel S: Mechanism of action and overview of vagus nerve stimulation technology, In Elijamel S, Slavin KV, editors:
Neurostimulation: Principles and Practice, Oxford, 2013, Wiley Blackwell, pp 111–120.

112. What should a teenager with epilepsy be told about the potential of obtaining a driver’s license?
State requirements vary regarding individuals with epilepsy and the right to drive. The most common requirement is a specified seizure-free period and submission of a physician’s evaluation of the patient’s ability to drive safely. Many states require the periodic submission of medical reports while the license is active. In addition, many states allow exceptions under which a license may be issued for a shorter seizure-free period (e.g., if a seizure occurred in isolation as a result of medication change or intercurrent illness), or they may issue licenses with restrictions (e.g., daytime driving only). A summary of requirements for each state is available from the Epilepsy Foundation.

Epilepsy Foundation: www.epilepsy.com. Accessed on March 5, 2015.

113. What are some common seizure triggers about which families should be counseled?
• Sleep deprivation, insufficient sleep, being overtired
• Fever and illnesses, particularly viral
• Low blood sugar, poor oral intake
• Flashing bright lights or patterns
• Association with menses
• Alcohol or drug use
• Stress
• Excess caffeine

Epilepsy Foundation: www.epilepsy.com/learn/triggers-seizures. Accessed on March 6, 2015.

114. When should a child be referred for possible epilepsy surgery? Although many epilepsy syndromes in childhood have spontaneous remission, 20% of incident epilepsy is intractable, and 5% of patients with intractable epilepsy may benefit from epilepsy surgery. Indications for surgery include failure of two AEDs, intractable disabling seizures, and/or deteriorating development. In general, outcome is determined by the completeness of the evaluation and the congruence of the data, the completeness of the resection, and the etiology of the seizures.

Ryvlin R, Cross JH, Rheims S: Epilepsy surgery in children and adults, Lancet Neurol 13:1114–1126, 2014.

FEBRILE SEIZURES
115. How are febrile seizures defined? Febrile seizures are defined as a convulsion caused by a fever (temperature > 100.4 °F or 38 °C by any method) that is without evidence of CNS pathology or acute electrolyte imbalance that occurs in children between the ages of 6 months and 60 months with a peak at the end of the second year of life). Children with a history of epilepsy who have an exacerbation of seizures with fever are excluded. Febrile seizures
occur in 2% to 5% of children. There is often a positive family history of febrile convulsions.
116. What is the likelihood of recurrence of a febrile seizure? The likelihood of recurrence increases with a younger age of onset, with a recurrence rate about 1 in 2 if the patient is <1 year of age when the initial seizure occurs and 1 in 5 if the patient is >3 years of age at the time of the initial seizure. About half of recurrences are within 6 months of the first seizure; three- fourths occur within 1 year, and 90% occur within 2 years. Other risk factors for recurrence are a lower temperature (close to 38 °C) at the time of seizure, <1 hour’s duration of fever before the seizure, and a family history of febrile seizures. Overall, the recurrence rate in the pediatric population is about 30%.

AAP Subcommittee on Febrile Seizures: Clinical practice guideline—febrile seizures: guideline for the neurodiagnostic evaluation of the child with a simple febrile seizure, Pediatrics 127:389–394, 2011.

117. What features make a febrile seizure complex rather than simple?
• Simple febrile seizure: Relatively brief (<15 minutes long) and occurs as a solitary event (one attack in 24 hours) in the setting of fever not caused by CNS infection
• Complex (also called atypical or complicated) febrile seizure: Focal features either at the onset or during the seizure, extended in duration (>15 minutes long), or occurring more than once in 1 day
118. Why are complex febrile seizures more worrisome than simple febrile seizures? They suggest a more serious problem. For example, a focal seizure raises concern of a localized or lateralized functional disturbance of the CNS. An unusually long seizure (>15 minutes) also raises the suspicion of primary CNS infectious, structural, or metabolic disease. Repeated seizures within a 24-hour period likewise imply a potentially more serious disorder or impending status epilepticus.
119. When should a lumbar puncture (LP) be performed as part of the evaluation of a child <12 months of age with a simple febrile seizure?
This had traditionally been a difficult question when a well-appearing infant or young toddler was
examined after a febrile seizure, but the widespread use of immunizations in the United States for two of the most common causes of bacterial meningitis, H. influenzae type b (Hib) and S. pneumoniae, has significantly lowered the incidence of bacterial meningitis. Current data no longer support a routine LP in a well-appearing, fully immunized child with a simple febrile seizure.
The American Academy of Pediatrics recommends a LP in any child who presents with a fever and seizure if the child has meningeal signs and symptoms (neck stiffness, Kernig and/or Brudzinski signs) or any history or exam suggestive of intracranial infection. An LP should be considered:
• If the patient is between 6 to 12 months of age and has not received scheduled immunizations (especially Hib and pneumococcal vaccinations) or when the immunization status cannot be determined. This child is at increased risk for bacterial meningitis. Patients
>12 months should have recognizable symptoms of bacterial meningitis.
• If a patient has been pretreated with antibiotics because antibiotic treatment can mask the
signs and symptoms of meningitis

AAP Subcommittee on Febrile Seizures: Clinical practice guideline—febrile seizures: guideline for the neurodiagnostic evaluation of the child with a simple febrile seizure, Pediatrics 127:389-394, 2011.

120. Are EEG or neuroimaging studies indicated for a child with a simple febrile seizure? No. An EEG done shortly after or within a month after a seizure does not predict either the recurrence of febrile seizures or the development of afebrile seizures/epilepsy in the ensuing 2 years. CT or MRI studies are not indicated because children who are neurologically healthy before a simple febrile seizure have a low likelihood of a clinically important intracranial structural abnormality.

AAP Subcommittee on Febrile Seizures: Clinical practice guideline—febrile seizures: guideline for the neurodiagnostic evaluation of the child with a simple febrile seizure, Pediatrics 127:389–394, 2011.

121. Do prolonged febrile seizures result in an increased peripheral white blood cell count?
A common clinical question in children is whether a leukocytosis, if found, can be explained on the basis of a prolonged seizure as a stress reaction. In a study of 203 children with seizures and fever, 61% had a normal peripheral white blood cell count. No association was found between blood leukocytosis and febrile seizure duration in children.

van Stuijvenberg M, Moll HA, Steyerberg EW, et al: The duration of febrile seizures and peripheral leukocytosis, J Pediatr
133:557–558, 1998.

122. What ancillary testing should be considered in a patient with a complex febrile seizure? Most children with their first complex febrile seizure should undergo a LP for a CSF examination to rule out intracranial infection. Children with focal motor seizures or postictal lateralized deficits (motor paresis, unilateral sensory or visual loss, sustained eye deviation, or aphasia) should be considered for emergent neuroimaging to exclude a structural abnormality before the LP. A LP could result in cerebral herniation if ICP is increased because of a mass effect. However, if the patient is neurologically normal, data suggest an emergent CT may not be necessary. The immediate performance of an EEG offers limited insight into the patient’s disease. Prominent generalized postictal slowing is not unexpected. Definite focal slowing suggests a possible structural abnormality. For a simple febrile seizure, an EEG is not indicated because it is not predictive of either the risk for recurrence of febrile seizures or the development of epilepsy.

Teng D, Dayan P, Tyler S, et al: Risk of intracranial pathologic conditions requiring emergency intervention after a first complex febrile seizure episode among children, Pediatrics 117:304–308, 2006.
DiMario FJ: Children presenting with complex febrile seizures do not routinely need computed tomography scanning in the emergency department, Pediatrics 117:528–530, 2006.

123. What is the risk for epilepsy after a simple febrile seizure? The risk depends on several variables. In otherwise normal children with a simple febrile seizure, the risk for later epilepsy is about 2%. The risk for epilepsy is higher if any of the following is present:
• There is a close family history of nonfebrile seizures.
• Prior neurologic or developmental abnormalities exist.
• The patient had an atypical or complex febrile seizure, defined as focal seizures, seizures lasting at least 15 minutes, and/or multiple attacks within 24 hours.
One risk factor increases the risk to 3%. If all three risk factors are present, the likelihood of later epilepsy increases to 5% to 10%.

Graves RC, Oehler K, Tingle LE: Febrile seizures: risks, evaluation, and prognosis, Am Fam Physician 85:149–153, 2012. Waruiru C, Appleton R: Febrile seizures: an update, Arch Dis Child 89:751–756, 2004.

KEY POINTS: FEBRILE SEIZURES
1. Simple: Brief and lasting <15 minutes
2. Complex: Focal, >15 minutes long, or recurrence within 1 day
3. Risk for recurrent febrile seizure increases if positive family history or seizure occurs at <1 year of age and/or body temperature of <40°C
4. Risk for developing future nonfebrile seizures is low (only 2% by age 7 years)
5. Normal long-term intellect and behavior compared with controls
6. Increased risk for developing epilepsy if complex febrile seizure, prior neurologic abnormality, or family history of seizure disorder

124. What is the long-term outcome for children with febrile seizures?
In a previously normal child, the risk for death, neurologic damage, or persistent cognitive impairment from a single benign febrile seizure is near zero. These potential complications are more likely with complex febrile seizures, but the risk is still exceedingly low. Impaired cognition in the latter group is

more likely if afebrile seizures subsequently develop. Febrile status epilepticus has a very low mortality with proper treatment. However, the potential development of hippocampal injury (mesial temporal sclerosis) is currently being evaluated in the United States in the FEBSTAT study.

Scott RC: Consequences of febrile seizures in childhood, Curr Opin Pediatr 26:662–667, 2014.
Verity CM, Greenwood R, Golding J: Long-term intellectual and behavioral outcomes of children with febrile convulsions,
N Engl J Med 338:1723–1728, 1998.

125. After a febrile seizure, should a child be treated with prophylactic AEDs?
For most children, a simple febrile seizure is an unwanted but transient disruption of their health, and treatment is not necessary. Treatment interventions, either continuous or intermittent (at the time of fever) have been evaluated for valproate, pyridoxine, phenobarbital, phenytoin, diazepam, and clonazepam. Adverse effects were noted in up to 30% in the phenobarbital group (lower comprehension scores) and up to 36% in the benzodiazepine treated group. Long-term prophylaxis does not improve the prognosis in terms of subsequent epilepsy or motor or cognitive ability. In general, the side effects of prophylaxis (especially the hepatotoxicity and pancreatopathy associated with valproic acid therapy) outweigh the relatively minor risks of recurrence. Exceptions could include the very young child if febrile seizures recur frequently, children with preexisting neurologic abnormalities or children with recurrent complex febrile seizures.

Offringa M, Newton R: Prophylactic drug management for febrile seizures in children, Cochrane Database Syst REV 18: CD003031, 2012.

126. Is the aggressive use of antipyretic therapy at the start of a febrile illness effective in reducing the likelihood of a febrile seizure? Despite being recommended frequently by pediatricians, aggressive antipyretic use has not been shown to be effective in preventing recurrence of a febrile seizure.

Offringa M, Newton R: Prophylactic drug management for febrile seizures in children, Cochrane Database Syst REV 18: CD003031, 2012.
Strengell T, Uhari M, Tarkka R, et al: Antipyretic agents for preventing recurrences of febrile seizures: randomized controlled trial, Arch Pediatr Adolesc Med 163:799–804, 2009.

HEADACHE
127. What are the emergency priorities when evaluating a child with a severe headache? As with all common presenting symptoms, the main priority is to rule out diagnostic possibilities that may be life-threatening:
• Increased ICP (e.g., mass lesion, acute hydrocephalus)
• Intracranial infections (e.g., meningitis, encephalitis)
• Subarachnoid hemorrhage
• Stroke
• Malignant hypertension
• Acute angle closure glaucoma (may appear as a headache, but rare in children)

128. When should neuroimaging be considered in a child with headache?
• Abnormal neurologic signs (oculomotor abnormalities, gait ataxia, papilledema, focal weakness)
• Headache increasing in frequency and severity
• Headache occurring in early morning or awakening child from sleep
• Headache made worse by straining or by sneezing or coughing (may be a sign of increased ICP)
• Headache associated with severe vomiting without nausea
• Headache worsened or helped significantly by a change in position
• Macrocephaly
• Fall off in linear growth rate
• Recent school failure or significant behavioral changes
• New-onset seizures, especially if seizure has a focal onset

• Migraine headache and seizure occurring in the same episode, with vascular symptoms preceding the seizure (20% to 50% risk for tumor or arteriovenous malformation)
• Cluster headaches in any child or teenager

Lewis DW, Ashwal S, Dahl G, et al, Quality Standards Subcommittee of the American Academy of Neurology; Practice Committee of the Child Neurology Society: Practice parameter: evaluation of children and adolescents with recurrent headaches. Report of the Quality Standards Subcommittee of the American Academy of Neurology and the Practice Committee of the Child Neurology Society, Neurology 59:490–498, 2002.

KEY POINTS: CLASSIC HEADACHE OF INCREASED INTRACRANIAL PRESSURE
1. Awakens patient from sleep at night
2. Pain present upon awakening in the morning
3. Vomiting without associated nausea
4. Made worse by straining, sneezing, or coughing
5. Intensity of pain changes with changes in body position
6. Pain lessens during the day

129. What are the three primary headache disorders in children?
These are recurrent headaches not attributable to underlying physical disease.
• Migraine: Most common type in children (4% in childhood, with a male predominance; after adolescence, more common in females)
• Tension type: Features different from migraine—bilateral, non-pulsating, not aggravated by activity; school problems with stress and absences and family dysfunction are frequently noted
• Cluster: Uncommon in childhood; consist of severe unilateral orbital or supraorbital pain with conjunctival injection and tearing

130. What are the clinical features of migraine headaches in children? Migraine is a periodic disorder with symptom-free periods characterized by headaches with a throbbing nature, unilateral in older children and commonly bilateral in younger children, lasting 1 to 72 hours, pulsating with moderate or severe intensity, aggravated by routine physical activity and exercise, and associated with nausea and/or photophobia and phonophobia. There may be a history of recurrent vomiting, motion sickness, or vertigo. There is often a family history of migraine, and the genetics may be multifactorial.
• Migraine with aura: Previously called classic migraine, this is less common in children. The aura is a prodrome of variable focal neurologic features such as visual scotoma, sensory symptoms (numbness, tingling), sluggishness, and difficulty concentrating or motor features (weakness, dysphasia).
• Migraine without aura: Previously called common migraine, these are the more frequent type in childhood.
Other clinical syndromes that are considered migraine variants in childhood include cyclic vomiting syndrome; abdominal migraine; benign paroxysmal vertigo of childhood; and possibly, infantile colic.

Headache Classification Subcommittee of the International Headache Society: The international classification of headache disorders, Cephalalgia 24(Suppl 1):1–160, 2004.

131. Which physical findings are important during the initial evaluation of possible migraine headache?
• Height and weight should be normal for age. Pituitary tumor, craniopharyngioma, and partial ornithine transcarbamylase deficiency may all result in growth failure and mimic migraine headache. Head circumference should be normal, ruling out hydrocephalus.
• Skin should be checked for abnormalities. Throbbing headaches are common in neurofibromatosis and systemic lupus erythematosus, both of which have easily recognizable skin manifestations.
• Blood pressure should be normal.

• Check for sinus tenderness or pain with sinus percussion or head movement (implying cervical spine disease). The patient should be examined for carious teeth, misaligned bite, or disordered chewing and jaw opening (temporomandibular joint dysfunction).
• Auscultation should reveal no cranial bruits (if present, these suggest possible arteriovenous malformation or mass lesion).
• The neurologic examination should be normal.
132. When do children begin to have migraine headaches?
About 20% suffer their first headache before the age of 10 years.
Infantile migraine does occur, and often manifests as vomiting, pallor, vertigo, and ataxia, with or without headache, which can occur in a periodic fashion and frequently improves with sleep.

Barlow CF: Migraine in the infant and toddler, J Child Neurol 9:92–94, 1994.

133. Which foods have been associated with the development of migraine headaches? Tyramine-rich foods (cheese, red wine), foods with monosodium glutamate (Asian food, adobo seasoning), nitrate-rich foods (smoked and lunch meats, salami), alcoholic beverages, caffeinated beverages, chocolate, citrus fruits, and sulfites (food coloring) have been associated with the development of migraine headaches.
134. What is familial hemiplegic migraine?
Familial hemiplegic migraine is an autosomal dominant disorder that is clinically characterized by transient hemiparesis and aphasia followed by migraine headache. About 20% are affected by progressive cerebellar ataxia. Mutations in CACNA1A (which encodes a neuronal calcium channel) on chromosome 19 are found in half of affected families.

Wessman M, Kaunisto MA, Kallela M, et al: The molecular genetics of migraine, Ann Med 36:462–473, 2004.

135. What is the likely diagnosis for a 10-year-old girl with a history of headaches and a family history of migraines who has had 10 minutes of a spinning sensation and double vision followed by an occipital headache and has a normal neurologic examination in the office?
Basilar-type migraine, which occurs in 3% to 19% of childhood migraines, is a likely diagnosis. Symptoms related to balance, gait, and visual disturbance are followed by headache, which, unlike most migraines, is occipital. Patients with basilar artery migraine may have drop attacks with altered awareness. Syncope is also more common in patients with migraine compared with the general population.

Lewis DW: Pediatric migraine, Pediatr REV 28:43–53, 2007.

136. How do the triptans work to treat an acute migraine headache?
Triptans are serotonin receptor subtype-selective drugs, which were thought initially to work primarily through their vasoconstrictive effects on arterial smooth muscle in cranial blood vessels. However, there are questions whether the primary mechanism is central or peripheral. Triptans act on peripheral nerve endings, preventing the release of proinflammatory and vasoactive peptides, including substance P and calcitonin gene–related peptide (GCRP). Also unclear is the apparent selectivity of triptans for migraine pain but not other kinds of somatic pain.

Pringsheim T, Becker WJ: Triptans for symptomatic treatment of migraine headache, BMJ 348:g2285, 2014.

137. What nonpharmacologic therapies are available for the prevention of migraine?
• Migraine elimination diet
• Vitamin therapy: riboflavin, coenzyme Q10, magnesium
• Normalization of sleep habits
• Discontinuance of possible triggering medications (e.g., analgesic overuse, bronchodilators, oral contraceptives)
• Biofeedback

• Relaxation therapy
• Family counseling (if family stress is a trigger)
• Self-hypnosis

Nicholson RA, Buse DC, Andrasik F, Lipton RB: Nonpharmacologic treatments for migraine and tension-type headache: how to choose and when to use, Curr Treat Options Neurol 13:28–40, 2011.

138. What categories of medication are available for the prevention of migraine in children?
As with many therapies used for children, most studies involve adults with extrapolation to children for whom the medications may not work as well. These medications are regularly used by clinicians but are not yet approved by the U.S. Food and Drug Administration for children. Keys to therapy
are gradually increasing the dose until effectiveness is or is not established or adverse effects intervene.
• Antidepressants (e.g., tricyclics such as amitriptyline)
• Antihistamine (e.g., cyproheptadine, which has antiserotonergic effects)
• Antihypertensives (e.g., β-blockers such as propranolol and calcium channel blockers)
• Anticonvulsants (including divalproex sodium and topiramate)

Damen L, Bruijn JK, Verhagen AP, et al: Prophylactic treatment of migraine in children. Part 2. A systematic review of pharmacological trials, Cephalalgia 26:373–383, 2006.

139. Who should be started on prophylactic medication for migraine headaches?
There are no precise criteria, but generally prophylactic treatment should be considered if any of the following are present:
• Headaches with aura occur frequently.
• Headaches with aura are poorly responsive to abortive medication.
• School attendance is significantly affected.
• Headaches, although infrequent, last for several days.
140. How long are the prophylactic medications continued?
The optimal duration of therapy remains unclear, but many authorities suggest a treatment duration of 3 to 6 months followed by an attempt at weaning. Less than 50% will require the reinitiation of medication.
141. What distinguishes tension-type headaches from migraines? Unlike migraines, these headaches are bilateral with a pressing and tightening quality (as opposed to the pulsatile quality of migraines) and usually of mild or moderate intensity. They are not associated with nausea or vomiting and typically not worsened by light or sound. Pericranial muscle tenderness is common.
Psychological stress is associated with and can aggravate tension-type headaches. Activation of hyperexcitable peripheral afferent neurons from head and neck muscles, as well as abnormalities in central pain processing and pain sensitivity, likely contributes to the problem.

Loder E, Rizzoli R: Tension-type headache, BMJ 336:88–92, 2008.

MOVEMENT DISORDERS
142. What are the various types of pathologic hyperkinetic movements?
• Tremors: Rhythmic oscillatory movements, both supination-pronation and flexion-extension, seen in resting state or with activity
• Chorea: Quick dancing movements of proximal and distal muscles with irregular unpredictable random jerks
• Athetosis: Irregular, slow, distal writhing movements
• Stereotypy: Repetitive, purposeless motions (e.g., body rocking, head rolling) that resemble voluntary movements often associated with akathisia (sensory and motor restlessness); often seen in autism spectrum disorders

• Dystonia: Slow, twisting, sustained movements; may result in abnormal postures and progress to contractures
• Ballismus: Abrupt, random, violent, flinging movements, often proximal and unilateral
• Myoclonus: Abrupt, brief, jerky contractions of one or more muscles, often stimulus sensitive
• Tics: Rapid, sudden, repetitive movements or vocalizations
143. What techniques can be used to elicit abnormal movements (particularly chorea)? Methods of provocative testing include the maintenance of posture in extension against gravity, hyperpronation (or “spooning,” especially above the head), tongue protrusion (“trombone tongue”), squeezing the finger of the examiner (“milk-maid’s grip”), pouring liquid, and drawing a spiral.
144. What disorders are commonly associated with the various hyperkinetic movements?
• Tremors, resting: Primary juvenile Parkinson disease, secondary Parkinson disease
• Tremors, kinetic: Essential (familial) tremor, cerebellar disorders, brainstem tumors, hyperthyroidism, Wilson disease, electrolyte disturbance (e.g., glucose, calcium, magnesium), heavy-metal intoxication (e.g., lead, mercury), multiple sclerosis
• Chorea: Sydenham chorea (associated with rheumatic fever), Huntington disease, hyperthyroidism, infectious mononucleosis, pregnancy, anticonvulsants, neuroleptic drugs, closed head injury, systemic lupus erythematosus, carbon monoxide poisoning, Wilson disease, hypocalcemia, polycythemia, parainfectious and infectious encephalopathies (e.g., rubeola, syphilis)
• Athetosis: CP, other static encephalopathies, Lesch-Nyhan syndrome, kernicterus
• Stereotypy: Autism, Rett syndrome, neuroleptic drugs (i.e., tardive dyskinesia), schizophrenia
• Dystonia: Idiopathic primary dystonias (e.g., torsion dystonia), Sandifer syndrome, spasmus nutans, neuroleptic drugs, static encephalopathy, perinatal asphyxia, familial dystonia (sometimes dopa-responsive)
• Ballismus: Encephalitis, closed head injury
• Myoclonus: Sleep myoclonus, benign myoclonus of infancy, postanoxic encephalopathy, uremic encephalopathy, hyperthyroidism, urea cycle defects, side effects of tricyclic therapy, slow virus infections, Wilson disease, myoclonus-opsoclonus, neuroblastoma, epileptic encephalopathies, mitochondrial disease, prion disease, Tay-Sachs disease, startle disease, sialidosis
145. What constitutes a tic? Tics are brief, sudden, repetitive, stereotyped, involuntary, and purposeless movements or vocalizations. They most commonly involve muscles of the head, neck, and respiratory tract. Their frequency can be increased by anxiety, stress, excitement, and fatigue. They are decreased during sleep and relaxation; during activities involving high concentration; and, at times, through voluntary action. In some cases, premonitory feelings (e.g., irritation, tickle, temperature change) can precipitate the motor or vocal response.
146. What is the range of clinical tics?
• Motor (simple clonic): Eye blinking, eye deviation, head twitching, shoulder shrugging
• Motor (simple dystonic): Bruxism, abdominal tensing, shoulder rotation
• Motor (complex): Grunting, barking, sniffing, snorting, throat clearing, spitting
• Vocal (complex): Coprolalia (obscene words), echolalia (repeating another’s words), palilalia (rapidly repeating one’s own words)
147. What are the causes of a tic? Transient and chronic tic disorders usually do not have an identifiable cause. However, dyskinesias such as tics can be found in association with a number of other conditions:
• Chromosomal abnormalities: Down syndrome, fragile X syndrome
• Developmental syndromes: Autism, pervasive developmental disorder, Rett syndrome
• Drugs: Anticonvulsants, stimulants (e.g., amphetamines, cocaine, methylphenidate, pemoline)
• Infections: Encephalitis, post-rubella syndrome
148. How should simple tics be treated?
Simple motor tics are common and occur in more than 5% to 21% of school-age children. Simple tics generally do not require pharmacologic intervention and can be treated expectantly by developing relaxation techniques, minimizing stresses that exacerbate the problem, avoiding punishment for tics,

and decreasing fixation on the problem. Most simple tics self-resolve in 2 to 12 months. Moderate or severe tics, especially when significant patient distress is involved, may warrant pharmacologic treatment.
149. What comorbidities occur in children with tics? The prevalence of tic disorder is higher in younger children and in males and is associated with school dysfunction, learning disabilities, obsessive-compulsive disorder, and attention-deficit/hyperactivity disorder. In addition, separation anxiety, overanxious disorder, simple phobia, social phobia, agoraphobia, mania, major depression, and oppositional defiant disorder were found to be significantly more common in children with tics.
150. When do tics warrant pharmacologic intervention? Tics that have a significant disabling impact on a child’s educational, social, or psychological well-being (particularly if they have been present for >1 year) may require intervention. When the complexity of tics increases or the diagnosis of Tourette syndrome is suspected, pharmacotherapy should also be
considered. Most theories point to a hyper-dopaminergic state of the basal ganglia as the most likely etiology for unregulated movements. Pharmacologic management includes α2-agonists (e.g., clonidine, guanfacine) or the administration of atypical neuroleptics (e.g., risperidone, haloperidol) and/or the cessation of any stimulant drugs that can cause dopamine release. Because of the high associated incidence of obsessive-compulsive disorder and attention-deficit/hyperactivity disorder, other medications may be needed, and consultation with a pediatric psychiatrist or neurologist is often warranted.
151. What are the diagnostic criteria for Tourette syndrome?
In 1885, Gilles de la Tourette described a syndrome of motor tics and vocal tics with behavioral disturbances and a chronic and variable course. Diagnostic and Statistical Manual of Mental Disorders (DSM-V) criteria for Tourette syndrome require the following:
• Two or more motor tics and at least one vocal tic
• Presence of tics for more than 1 year (usually on a daily basis, but can be intermittent)
• Onset before the age of 18 years
• Not caused by medications or any identifiable medical etiology

American Psychiatric Association: Diagnosis and Statistical Manual of Mental Disorders, ed 5, Washington, DC, 2013, American Psychiatric Association.

152. What is coprolalia?
Coprolalia is an irresistible urge to utter profanities, occurring as a phonic tic. Only 20% to 40% of patients with Tourette syndrome have this phenomenon, and it is not essential for the diagnosis.
153. What behavioral problems are associated with Tourette syndrome?
• Obsessive-compulsive disorder
• Attention-deficit/hyperactivity disorder
• Severe conduct disorders
• Learning disabilities (particularly math)
• Sleep abnormalities
• Depression, anxiety, and emotional lability

Tourette Syndrome Association: www.tsa-usa.org. Accessed on March 6, 2015.
Robertson MM: The Gilles De La Tourette syndrome: the current status, Arch Dis Child 97:166–175, 2012.

154. Why is the diagnosis of Tourette syndrome commonly delayed?
• Tendency to associate unusual symptoms with attention-getting or psychological problems
• Incorrect belief that all children with Tourette syndrome must have severe tics
• Attribution of vocal tics to upper respiratory infections, allergies, or sinus or bronchial problems
• Diagnosis of eye blinking or ocular tics as ophthalmologic problems
• Mistaken belief that coprolalia is an essential diagnostic feature

Singer HS: Tic disorders, Pediatr Ann 22:22–29, 1993.

155. What is the cause of tardive dyskinesia? TARDIVE dyskinesia is a hyperkinetic disorder of abnormal movements, most commonly involving the face (e.g., lip smacking or pursing, chewing, grimacing, tongue protruding). Tardive dyskinesia occurs during treatment with neuroleptics (e.g., chlorpromazine, haloperidol, metoclopramide) or within 6 months of their discontinuance. This disorder is thought to be a result of dopaminergic dysfunction of the basal ganglia because these drugs act as dopamine-receptor blockers.
156. For a patient taking neuroleptic medication, how long must therapy last before symptoms of tardive dyskinesia can develop?
About 3 months of continuous or intermittent treatment with neuroleptics is needed before the risk for tardive dyskinesia increases.
157. What is neuroleptic malignant syndrome?
Neuroleptic malignant syndrome is a syndrome of movement (rigidity, tremor, chorea, and dystonia), autonomic dysfunction (fever, hypertension, tachycardia, diaphoresis, irregular respiratory pattern, urinary retention), alteration of consciousness, and rhabdomyolysis with an elevation of creatinine kinase. It occurs within weeks of starting neuroleptics, and there is a 20% associated mortality rate in adults.
158. Which movement disorder in children presents with “dancing eyes and dancing feet”?
Opsoclonus-myoclonus (infantile polymyoclonus syndrome or acute myoclonic encephalopathy of infants) is a rare but distinctive movement disorder in children that is seen during the first 1 to 3 years of life. Opsoclonus is characterized by wild, chaotic, fluttering, irregular, rapid, conjugate bursts of eye movements (saccadomania). Myoclonus is sudden, shocklike muscular twitches of the face, limbs, or trunk. The anatomic site of pathology is the cerebellar outflow tracts. The etiology may be direct viral invasion, postinfectious encephalopathy, or neuroblastoma. Immunomodulatory therapy with corticosteroids (ACTH, dexamethasone), intravenous immunoglobulin (IVIG), and rituximab may be useful.

NEONATAL SEIZURES
159. How are neonatal seizures classified? Although there is no universally accepted standard classification system, one based on clinical criteria is commonly used. It divides neonatal seizures into four types:
• Subtle (ocular phenomena, oro-buccal-lingual movements, limb bicycling, autonomic phenomena, apneic seizures)
• Tonic (focal or generalized)
• Clonic (focal or multifocal)
• Myoclonic (focal, multifocal, or generalized)
All seizure types are recognized as paroxysmal alterations in behavioral, motor, or autonomic function. Not all clinically observed phenomena, however, are accompanied by associated epileptic surface-EEG activity, and this electroclinical disassociation is increased after AED treatment. Partial clonic, tonic, and myoclonic seizures have been shown to have the most consistent EEG ictal correlate.
160. Why are generalized seizures uncommon in newborns?
Generalized seizures are rarely seen in neonates because of incomplete myelination, which tends to prevent highly organized, synchronized ictal motor activity from occurring.
161. What is the most common type of clinical seizure during the neonatal period?
The so-called subtle seizure is the most common. Rather than arising as an abrupt dramatic “convulsion” with obvious forceful twitching or posturing of the muscles, the subtle seizure appears as unnatural, repetitive, stereotyped choreography, featuring oral-buccal-lingual movements, eye blinking, nystagmus, lip smacking, or complex integrated limb movements (swimming, pedaling, or rowing) and other fragments of activity drawn from the limited repertoire of normal infant activity. These neonates frequently have HIE and moderately to markedly abnormal EEGs, and they are at significantly greater risk for mental retardation, CP, and epilepsy.
162. What are the causes of neonatal seizures?
• Hypoxic-ischemic encephalopathy caused by birth asphyxia
• Infection
• Toxins (e.g., inadvertent fetal injection with local anesthetic; cocaine, including withdrawal)

• Metabolic abnormalities (e.g., hypoglycemia, hypocalcemia, hypomagnesemia, pyridoxine deficiency, inborn errors)
• CNS malformations
• Cerebrovascular lesions (e.g., intraventricular, periventricular hemorrhage, subarachnoid hemorrhage, infarction, arterial cerebral occlusion)
• Benign familial neonatal-infantile seizures (e.g., a sodium channelopathy)

Glass HC: Neonatal seizures: advances in mechanisms and management, Clin Perinatol 41:177–190, 2014. Zupanc ML: Neonatal seizures, Pediatr Clin North Am 51:961–978, 2004.

163. In premature and full-term infants, how do the causes of seizures vary with regard to relative frequency and time of onset?
See Table 13-4.

Table 13-4. Variance in Relative Frequency and Time of Onset of Causes of Seizures

ETIOLOGY Postnatal
0-3 DAYS Time of Onset
>3 DAYS Relative
PREMATURE Frequency
FULL-TERM
Hypoxic-ischemic + +++ +++
Intracranial hemorrhage*
+ + ++ +
Hypoglycemia + + +
Hypocalcemia + + + +
Intracranial infection†
+ + ++ +
Developmental defects + + ++ ++
Drug withdrawal + + + +
*Hemorrhages are principally germinal matrix-intraventricular in the premature infant and subarachnoid or subdural in the term
infant.
†Early seizures occur usually after intrauterine nonbacterial infections (e.g., toxoplasmosis, cytomegalovirus infection), and later seizures usually occur with herpes simplex encephalitis or bacterial meningitis.
Adapted from Volpe JJ, editor: Neurology of the Newborn, ed 3. Philadelphia, 1995, WB Saunders, p 184.

164. What is an acceptable workup in a newborn with seizures?
• A careful prenatal and natal history and a complete physical examination are needed.
• Laboratory studies should include blood for glucose, electrolytes, calcium, phosphorus, and magnesium.
• A lumbar puncture should be performed to rule out meningitis.
• Neuroimaging studies (cranial ultrasound, CT scan, or preferably, MRI) are mandatory.
• Additional studies may include blood levels for ammonia, lactate, and pyruvate; additional CSF studies (e.g., lactate, pyruvate, glucose, glycine, CSF neurotransmitters if metabolic disease is suspected); and urine studies for organic and amino acid analysis for possible inborn errors of metabolism.
• Serial use of EEG polygraphy can document persistent seizures, especially the persistence of electrographic seizures without clinical seizures after initial treatment.
165. In what settings should an inborn error of metabolism be suspected as a cause of neonatal seizures?
• The onset of seizures is beyond day 1 of life. (The exception is pyridoxine deficiency, which can occur on day of life 1; patients may have a history of seizures in utero.)
• The infant becomes symptomatic after the introduction of enteral or parenteral nutrition.
• The seizures are intractable and do not respond to conventional AEDs.
Characteristic EEG patterns may be seen in maple syrup urine disease, propionic acidemia, and pyridoxine deficiency.

Scher MS: Neonatal seizures. In Polin RA, Yoder MC, editors: Workbook in Practical Neonatology, ed 4. Philadelphia, 2007, Saunders Elsevier, p 363.

166. How are seizures differentiated from tremors in the neonate?
See Table 13-5.

Table 13-5. Tremors Versus Seizures
CLINICAL FEATURE TREMORS SEIZURES
Suppressibility + 0
Abnormality of gaze or eye movement 0 +
Movements are exquisitely stimulus sensitive + 0
Predominant movement Tremor Clonic jerking
Movements cease with passive flexion + 0
Autonomic changes 0 +

167. What are the treatment options for neonatal seizures? Neonatal seizures may be treated with phenobarbital. Studies of the pharmacokinetics of phenobarbital in neonates have indicated that it is most appropriate to load with a full 20 mg/kg rather than smaller fractions. If seizures persist, additional increments of phenobarbital to total loading doses of 40 mg/kg can be given. Continued seizures may be treated with a loading dose of 20 mg/kg of phenytoin (or phenytoin equivalents in the case of fosphenytoin). The usual maintenance dose for phenobarbital is between 3 and 6 mg/kg per day and between 4 and 8 mg/kg per day for phenytoin. Efficacy from either of these two agents is low, with only one-third of patients showing an immediate complete response. Even after apparently successful intravenous treatment with phenobarbital and phenytoin with the resolution of clinical seizures, electrographic seizures may continue unabated. The significance of this finding is unclear, and the need to suppress electrographic seizures without clinical accompaniments is controversial.
Other agents, such as levetiracetam and topiramate, have some evidence for efficacy, but are not at this time considered first-line agents for neonatal seizures. Lidocaine and midazolam have also been used in the treatment of neonatal status epilepticus.

Slaughter LA, Patel AD, Slaughter JL: Pharmacological treatment of neonatal seizures: a systematic review, J Child Neurol 28: 351–364, 2013.
Abend NS, Gutierrez-Colina AM, Monk HM, et al: Levetiracetam for treatment of neonatal seizures, J Child Neurol 26:465- 470, 2011.
Glass HC, Poulin C, Shevell MI: Topiramate for the treatment of neonatal seizures, Pediatr Neurol 244:439–442, 2011.

168. What is the treatment for refractory seizures in the neonate?
Frequent and recurrent seizures are not uncommon in newborns and are especially common in the setting of asphyxia. If seizures are refractory to full dosing of phenobarbital and phenytoin, the addition of drugs in the benzodiazepine family (e.g., diazepam, lorazepam, midazolam) is generally effective. It is important to ensure that no underlying biochemical disturbance is present before the serum levels of anticonvulsants are raised to maximal concentrations. Although pyridoxine-dependent seizures are rare, a trial dose of pyridoxine should be administered intravenously to infants with recurrent seizures of uncertain etiology. If possible, simultaneous EEG recordings should be performed to document the cessation of seizure activity and the normalization of the EEG within minutes of pyridoxine treatment. Infants with pyridoxine-dependent epilepsy may have profound autonomic dysfunction (apnea, bradycardia, and hypotension) in response to initial pyridoxine administration and should be monitored carefully. Folinic acid is allelic to pyridoxine deficiency and pyridoxal phosphate (P5P) is used in pyridoxine-resistant seizures.
169. Of what prognostic value is the interictal EEG in a neonate with seizures?
This study can have significant prognostic value. Severe interictal EEG abnormalities (e.g., burst suppression, marked voltage suppression, flat or isoelectric) are highly predictive (90%) of a fatal outcome or severe neurologic sequelae. Conversely, a normal interictal EEG in a term infant with

seizures confers a very low (10%) likelihood of significant neurologic impairment. Moderate abnormalities (e.g., voltage asymmetries, immature patterns) have a mixed outcome.

Laroia N, Guillet R, Burchfiel J, McBride MC: EEG background as predictor of electrographic seizures in high risk neonates,
Epilepsia 39:545–551, 1998.

170. After an infant has recovered from a seizure, how long should medication be continued?
There are no clear guidelines for duration of therapy after neonatal seizures. Maintenance therapy typically involves the use of phenobarbital because it is difficult to achieve therapeutic levels of phenytoin with oral administration in infancy, and others are less well studied. Although phenobarbital is generally well tolerated, it may have deleterious effects on behavior, attention span, and possibly brain development. It does not prevent the later development of epilepsy. Many authorities recommend discontinuing therapy if the neurologic examination has normalized. In addition, if the neurologic examination is abnormal but an EEG by the age of 3 months reveals no seizure activity, consideration can also be given to stopping phenobarbital.
171. In patients with neonatal seizures, how does the cause affect the prognosis?
See Table 13-6.

Table 13-6. Relationship Between Cause and Prognosis of Neonatal Seizure

ETIOLOGY FAVORABLE OUTCOME*

MIXED OUTCOME UNFAVORABLE OUTCOME*

Toxic-
metabolic Simple late-onset hypocalcemia Hypomagnesemia
Hyponatremia Mepivacaine toxicity Hypoglycemia
Early-onset complicated hypocalcemia
Pyridoxine dependency Some aminoacidurias
Asphyxia — Mild hypoxic-ischemic encephalopathy Severe hypoxic-ischemic encephalopathy
Hemorrhage Uncomplicated subarachnoid hemorrhage Subdural hematoma Intraventricular hemorrhage
(grades I and II) Intraventricular hemorrhage (grades III and IV)
Infection — Aseptic meningoencephalitis; some bacterial meningitides Herpes simplex encephalitis; some bacterial meningitides
Structural — Simple traumatic contusion Malformations of the central nervous system
*Favorable prognosis implies at least an 85% to 90% chance of survival and subsequent normal development. Unfavorable prognosis implies a high likelihood (85% to 90%) of death or serious handicap in survivors.
From Scher MS: Neonatal seizures. In Polin RA, Yoder MC, editors: Workbook in Practical Neonatology, ed 4. Philadelphia, 2007, Saunders ELSEVIER, p 370.

NEUROCUTANEOUS SYNDROMES
172. What are the three most common neurocutaneous syndromes? Neurocutaneous syndromes (also called phakomatoses) are disorders characterized by the presence of tumors in various parts of the body (including the ocular and central nervous systems) and characteristic dermatologic findings of varying severity. The three most common are
• Neurofibromatosis
• Tuberous sclerosis complex
• Sturge-Weber syndrome
173. What are the inheritance patterns of the various neurocutaneous syndromes?
• Neurofibromatosis: Autosomal dominant
• Tuberous sclerosis complex: Autosomal dominant

• von Hippel-Lindau syndrome: Autosomal dominant
• Incontinentia pigmenti: X-linked dominant
• Sturge-Weber syndrome: Sporadic
• Klippel-Trénaunay-Weber syndrome: Sporadic
174. What are the diagnostic criteria for neurofibromatosis-1 (NF1)?
Two or more of the following:
• Café-au-lait spots (6 or more that are >0.5 cm in diameter before puberty; 6 or more that are
>1.5 cm in diameter after puberty)
• Skinfold freckling (axillary or inguinal region)
• Neurofibromas (two or more) of any type, or at least one plexiform neurofibroma
• Iris hamartomas, also called Lisch nodules (two or more)
• Characteristic osseous lesion (i.e., sphenoid dysplasia, thinning of the cortex of the long bones with or without pseudoarthrosis)
• First-degree relative with NF1 diagnosed by the above criteria

Williams VC, Lucas J, Babcock MA, et al: Neurofibromatosis type 1 revisited, Pediatrics 123:124–133, 2009.

175. How does NF1 differ from NF2?
NF1, which is also known as classic VON Recklinghausen disease, is much more common (1 in every 3000 to 4000 births) than NF2 and accounts for up to 90% of cases of neurofibromatosis. NF2 (1 in every 50,000 births) is characterized by bilateral acoustic neuromas, intracranial and intraspinal tumors, and affected first-degree relatives. NF1 has been linked to alterations on chromosome 17, whereas NF2 is linked to alterations on chromosome 22. Dermatologic findings and peripheral neuromas are rare in NF2. Other rarer subtypes of neurofibromatoses (e.g., segmental distribution) have been described.

Asthagiri AR, Parry DM, Butman JA, et al: Neurofibromatosis type 2, Lancet 373:1974–1986, 2009.

176. How common are café-au-lait spots at birth?
Up to 2% of black infants will have three café-au-lait spots at birth, whereas one café-au-lait spot occurs in only 0.3% of white infants. White infants with multiple café-au-lait spots at birth are more likely than black infants to develop neurofibromatosis. In older children, a single café-au-lait spot that is more than 5 mm in diameter can be found in 10% of white and 25% of black children.

Hurwitz S: Neurofibromatosis. In Hurwitz S, editor: Clinical Pediatric Dermatology, ed 2. Philadelphia, 1993, WB Saunders, pp 624–629.

177. If a 2-year-old child has seven café-au-lait spots that are larger than 5 mm in diameter, what is the likelihood that neurofibromatosis will develop, and how will it evolve? Up to 75% of these children, if followed sequentially, will develop one of the varieties of neurofibromatosis, most commonly type 1. In a study of nearly 1900 patients, 46% with sporadic NF1 did not meet criteria by the age of 1 year. By the age of 8 years, however, 97% met the criteria, and by the age of 20 years, 100% did. The typical order of appearance of features is café-au-lait spots, axillary freckling, Lisch nodules, and neurofibromas. Yearly evaluation of patients with suspicious findings should include a careful skin examination, ophthalmologic evaluation, and blood pressure measurement.

DeBella K, Szudek J, Friedman JM: Use of the National Institutes of Health criteria for the diagnosis of neurofibromatosis 1 in children, Pediatrics 105:608–614, 2000.
Korf BR: Diagnostic outcome in children with multiple café-au-lait spots, Pediatrics 90:924–927, 1992.

178. What are Lisch nodules?
Pigmented iris hamartomas (Fig. 13-5). Although these are not usually present at birth in patients with NF1, up to 90% will develop multiple Lisch nodules by the age of 6 years. Hamartomas are focal malformations that are microscopically composed of multiple tissue types, and these can resemble neoplasms. However, unlike neoplasms, they grow at similar rates as normal components and are unlikely to pathologically compress adjacent tissue.

Figure 13-5. Pigmented iris hamartomas (Lisch nodules). (From Habif TP, editor: Clinical Dermatology: A Color Guide to Diagnosis and Therapy, ed 5. Philadelphia, 2010, ELSEVIER,
p 985.)

179. How common is a positive family history in cases of NF1? Because of the high spontaneous mutation rate for this autosomal dominant disease, only about 50% of newly diagnosed cases are associated with a positive family history.
180. What are the primary diagnostic criteria for tuberous sclerosis complex (TSC)? TSC is characterized by hamartomatous growths that occur in multiple tissues. The National Institutes of Health Consensus Conference in 1998 revised the diagnostic criteria for TSC on the basis of major or minor features. Definite TSC consisted of two major features or one major and two minor features; probable and possible TSC had fewer features (Table 13-7). No single finding was considered pathognomonic for TSC. Two gene site abnormalities, TSC1 (chromosome 9) and TSC2 (chromosome 16), have been identified. Genetic testing is now available.

Crino PB, Nathanson KL, Henske EP: The tuberous sclerosis complex, N Engl J Med 355:1345–1356, 2006.

Table 13-7. Diagnostic Features for Tuberous Sclerosis Complex
MAJOR FEATURES MINOR FEATURES
Facial angiofibromas Dental enamel pits
Nontraumatic ungual or periungual fibroma Bone cysts
Hypomelanotic macules (>3) Hamartomatous rectal polyps
Shagreen patch Gingival fibromas
Multiple retinal nodular hamartomas Cerebral white matter migration tracts
Cortical tuber
Subependymal nodule or giant cell astrocytoma
Cardiac rhabdomyoma, single or multiple

181. What is the classic triad of TSC?
• Seizures
• Mental retardation
• Facial angiofibroma (adenoma sebaceum)
However, less than one-third of patients will develop these classic features.

Staley BA, Vail EA, Thiele EA: Tuberous sclerosis complex: diagnostic challenges, presenting symptoms, and commonly missed signs, Pediatrics 127:e117–e125, 2011.

182. What is the most common presenting symptom of TSC? Seizures. About 85% of patients have seizures, and epileptic (previously called infantile) spasms are the most common. The first-line treatment of epileptic spasms in TSC is vigabatrin (as opposed to ACTH in other etiologies of epileptic spasm). Tonic and atonic seizures are also seen. Complex partial seizures are frequently seen in conjunction with other seizure types. Mental retardation is especially common with the onset of seizures before the age of 2 years. Autism and other behavioral disturbances are also frequently seen in children with TSC.

Staley BA, Vail EA, Thiele EA: Tuberous sclerosis complex: diagnostic challenges, presenting symptoms, and commonly missed signs, Pediatrics 127:e117–e125, 2011.
Curatolo P, Bombardieri R, Jozwiak S: Tuberous sclerosis, Lancet 372:657–668, 2008.

183. What are skin findings in patients with tuberous sclerosis?
See Table 13-8.

Table 13-8. Skin Findings in Tuberous Sclerosis
AGE AT ONSET SKIN FINDINGS INCIDENCE (%)
Birth or Later Hypopigmented macules 80
2-5 yr Angiofibromas 70
2-5 yr Shagreen patches 35
Puberty Periungual and gingival fibromas 20-50
Birth or Later Café-au-lait spots 25

184. Why is the term adenoma sebaceum a misnomer when used to describe patients with tuberous sclerosis?
On biopsy, these papules are actually angiofibromas. They have no connection to sebaceous units or adenomas. This rash occurs in about 75% of patients with tuberous sclerosis, usually developing on the nose and central face between the ages of 5 and 13 years. It is red, papular, and monomorphous, and it is often mistaken for acne (Fig. 13-6). The diagnosis of tuberous sclerosis should be entertained in children who develop a rash that is suggestive of acne well before puberty.

Figure 13-6. Adenoma sebaceum in patient with tuberous sclerosis. These angiofibromas first appear as flat, pink macules and later become papular. Lesions may bleed easily. (From Habif TP: Clinical Dermatology: A Color Guide to Diagnosis and Therapy, ed 5. Philadelphia, 2010, ELSEVIER, p 988.)

185. What is the “tuber” of tuberous sclerosis?
These 1- to 2-cm lesions consist of small stellate neurons and astroglial elements that are thought to be primitive cell lines resulting from abnormal differentiation. They may be located in various cortical regions. They are firm to the touch, like a small potato or tuber.

186. What is the tissue type of a shagreen patch?
A shagreen patch is an area of cutaneous thickening with a pebbled surface that, on biopsy, is a connective tissue nevus. The term shagreen derives from a type of leather that is embossed by knobs during the course of processing.

187. Which types of facial port-wine stains are most strongly associated with ophthalmic or CNS complications?
Port-wine stains can occur as isolated cutaneous birthmarks or, particularly in the areas underlying the birthmark, in association with structural abnormalities in the following areas: (1) the choroidal vessels of the eye, thereby leading to glaucoma; (2) the leptomeningeal vessels of the brain, thus leading to seizures (Sturge-Weber syndrome); and (3) hemangiomas in the spinal cord (Cobb syndrome).
Glaucoma or seizures are most often associated with port-wine stains in children demonstrating the following:
• Involvement of the eyelids
• Bilateral distribution of the birthmark
• Unilateral involvement of all three branches (V1, V2, V3) of the trigeminal nerve
• Ophthalmologic assessment and radiologic studies (CT or MRI) are indicated for children exhibiting these findings.

Sudarsanam A, Ardern-Holmes SL: Sturge-Weber syndrome: from the past to the present, Eur J Paediatr Neurol 18: 257–266, 2014.
Tallman B, Tan OT, Morelli JG, et al: Location of port-wine stains and the likelihood of ophthalmic and/or central nervous system complications, Pediatrics 87:323–327, 1991.

188. What are the three stages of incontinentia pigmenti?
Incontinentia pigmenti is an X-linked dominant disorder that is associated with seizures and mental retardation. Ectodermal tissues, such as eyes, nails, hair, and teeth, are also affected. The condition is presumed to be lethal to boys in utero because nearly 100% of cases are female. There are rare cases of XY patients with incontinentia pigmenti. It is caused by mutations in the NEMO (NF-kappaB essential modulator) gene, which is involved in cellular signal transduction.
• Stage 1—Vesicular stage: Lines of blisters are present on the trunk and extremities of the newborn that disappear in weeks or months. They may resemble herpetic vesicles. Microscopic examination of the vesicular fluid demonstrates eosinophils.
• Stage 2—Verrucous stage: Lesions develop in the patient at about 3 to 7 months of age that are brown and hyperkeratotic, resembling warts; these disappear over 1 to 2 years.
• Stage 3—Pigmented stage: Whorled, swirling (marble cake–like), macular, hyperpigmented lines develop. These may fade over time, leaving only remnant hypopigmentation in late adolescence or adulthood (which is sometimes considered a fourth stage).

189. What is the likely diagnosis for a 7-year-old who is noted to have recurrent nosebleeds, cutaneous telangiectasias on his lips, and an intracranial arteriovenous malformation on MRI?
This child has hereditary hemorrhagic telangiectasia, which has also been known as Osler-Weber- Rendu disease. This condition may affect up to 1 in 5000 in the United States. The condition consists of nosebleeds; skin, lip, and oral mucosal lesions (Fig. 13-7); visceral manifestations due to arteriovenous malformations in the lung, liver, gastrointestinal tract, and CNS; and a positive family history. Genetic mutations involve transforming growth factor-β, which causes abnormalities in blood vessel formation.

Giordano P, Lenato GM, Lastella P, et al: Hereditary hemorrhagic telangiectasia: arteriovenous malformations in children,
J Pediatr 163:179–183, 2013.

Figure 13-7. Hereditary hemorrhagic telangiectasia with lip telangiectasia. (From Habif TP, editor: Clinical Dermatology: A Color Guide to Diagnosis and Therapy, ed 5. Philadelphia, 2011, ELSEVIER, p 911.)

NEUROMUSCULAR DISORDERS
190. How can the anatomic site responsible for muscle weakness be determined clinically?
See Table 13-9.

Table 13-9. Clinical Determination of Anatomic Site Responsible for Muscle Weakness
UPPER MOTOR NEURON
ANTERIOR HORN CELL NEURO- MUSCULAR JUNCTION
PERIPHERAL NERVE

MUSCLE
Tone Increased (may be decreased acutely) Decreased Normal,
variable Decreased Decreased
Distribution Pattern (e.g., Variable, Fluctuating, Nerve Proximal> distal
hemiparesis, asymmetric cranial distribution
paraparesis) nerve
Distal> proximal involvement
Reflexes Increased (may be decreased early Decreased to absent Normal (unless severely involved) Decreased to absent Decreased
Babinski Extensor Flexor Flexor Flexor Flexor
Other Cognitive dysfunction, atrophy only very late Fasciculations, atrophy, no sensory involvement Fluctuating course Sensory nerve involvement, atrophy, rare fasciculations No sensory deficits; may be tenderness and signs of inflammation
Adapted from Packer RJ, Berman PH: Neurologic emergencies. In Fleisher GR, Ludwig S, editors: Textbook of Pediatric Emergency Medicine, ed 3. Baltimore, 1993, Williams & Wilkins, p 584.

191. What are the causes of acute generalized weakness?
• Infectious and postinfectious conditions: Acute infectious myositis, GBS, enteroviral infection
• Metabolic disorders: Acute intermittent porphyria, hereditary tyrosinemia
• Neuromuscular blockade: Botulism, tick paralysis
• Periodic paralysis: Familial (hyperkalemic, hypokalemic, normokalemic)

Fenichel GM: Clinical Pediatric Neurology: Signs and Symptoms Approach, ed 5. Philadelphia, 2009, Elsevier, p 197.

192. If a child presents with weakness, what aspects of the history and physical examination suggest a myopathic process?
History
• Gradual rather than sudden onset
• Proximal weakness (e.g., climbing stairs, running) rather than distal weakness (more characteristic of neuropathy) predominates
• Absence of sensory abnormalities, such as “pins-and-needles” sensations
• No bowel and bladder abnormalities
Physical examination
• Proximal weakness is greater than distal weakness (except in myotonic dystrophy)
• Positive Gowers sign (see question 193)
• Neck flexion weaker than neck extension
• During the early stages, reflexes normal or only slightly decreased
• Normal sensory examination
• Muscle wasting but no fasciculations
• Muscle hypertrophy seen in some dystrophies

Weiner HL, Urion DK, Levitt LP: Pediatric Neurology for the House Officer, Baltimore, 1988, Williams & Wilkins, pp 136–138.

193. What is the significance of a Gowers sign?
Weakness of truncal and proximal lower extremity muscles. Most classically seen in Duchenne muscular dystrophy, the sign describes the manner in which children turn prone to rise and then rise from a sitting position by grasping and pushing on the knees and thighs (“climbing up the thighs”) until they are standing (Fig. 13-8). The adaptation of a prone position before rising is an important early feature because only 6.5% of healthy children still roll prone before standing. After age 3 years, any child with a need to turn prone before rising should be followed closely for a possible underlying neuromuscular condition.

Wallace GB, Newton RW: Gower’s sign revisited, Arch Dis Child 64:1317–1319, 1989.

Figure 13-8. Gowers sign. (a) A child turns prone to rise and begins to use his hands to push his body upright, (b) finally pushing off his knees/thighs before standing (From Lissauer T, Clayden G, Craft A: Illustrated Textbook of Paediatrics, ed 4. London, 2012, ELSEVIER Ltd, p 488.)

194. How does electromyography help differentiate between myopathic and neurogenic disorders?
Electromyography measures the electrical activity of resting and voluntary muscle activity. Normally, the action potentials are of standardized duration and amplitude, with two to four distinguishable phases. In myopathic conditions, the durations and amplitudes are shorter than expected, called brief,

small-amplitude potentials (BSAPs); in neuropathies, they are longer. In both conditions, extra phases (i.e., polyphasic units) are usually noted.
195. How is pseudoparalysis distinguished from true neuromuscular disease? Pseudoparalysis (hysterical paralysis) or weakness may be seen in conversion reactions (i.e., emotional conflicts presenting as symptoms). In conversion reactions, sensation, deep tendon reflexes, and Babinski response are normal; movement may also be noted during sleep. HOOVER sign is also helpful in cases of unilateral paralysis. With the patient lying supine on the table, the examiner places a hand under the heel of the affected limb and asks the patient to raise the unaffected limb. In pseudoparalysis, the examiner will feel pressure on the hand as the patient involuntarily extends the weak hip.
196. Why is it important to localize the cause of hypotonia? Localization of the level of the lesion is critical for determining the nature of the pathologic process. In the absence of an acute encephalopathy, the differential diagnosis of hypotonia is best approached by asking the question, “Does the patient have normal strength despite the hypotonia, or is the patient weak and hypotonic?” The combination of weakness and hypotonia usually points to an abnormality of the anterior horn cell or the peripheral neuromuscular apparatus, whereas hypotonia with normal strength is more characteristic of brain or spinal cord disturbances.

KEY POINTS: HYPOTONIA
1. Localization of lesion is critical for determining pathologic process.
2. Most important question: Is strength normal or abnormal?
3. Hypotonia with weakness: Think abnormality in anterior horn cell or peripheral neuromuscular apparatus.
4. Hypotonia without weakness: Think brain or spinal cord disturbance.

197. How can you detect myotonia clinically?
Myotonia is a painless tonic spasm of muscle that follows voluntary contraction, involuntary failure of relaxation, or delayed muscle relaxation after a contraction. It can be elicited by grip (e.g., handshake), forced eyelid closure (or delayed eye opening in crying infants), lid lag after upward gaze, or percussion over various sites (e.g., thenar eminence, tongue).
198. How do the presentations of the two forms of myotonic dystrophy differ? The presentation of congenital myotonic dystrophy is during the immediate newborn period. Symptoms include hypotonia; facial diplegia with “tenting” of the upper lip; and, frequently, severe respiratory distress as a result of intercostal and diaphragmatic weakness, especially in the right hemidiaphragm. Feeding problems as a result of poor suck and gastrointestinal dysmotility are also present. The juvenile presentation of this condition is during the first decade of life. This form is characterized by progressive weakness and atrophy of the facial and sternocleidomastoid muscles and shoulder girdle, impaired hearing and speech, and excessive daytime sleepiness. Clinical myotonia is more likely, and there may be mental retardation.
199. In a newborn with weakness and hypotonia, what obstetric and delivery features suggest a diagnosis of congenital myotonic dystrophy?
A history of spontaneous abortions, polyhydramnios, decreased fetal movements, delays in second-stage labor, retained placenta, and postpartum hemorrhage all raise the concern for congenital myotonic dystrophy. Because the mother is nearly always affected in congenital myotonic dystrophy (although previously diagnosed in only one-half of the cases), a careful clinical and electromyographic evaluation of the mother is essential. It is always important to shake the hand of the mother (barring religious exclusions) because affected women may not be able to release their hand after a handshake.
200. How is myotonic dystrophy an example of the phenomenon of “anticipation”? Genetic studies have shown that the defect in myotonic dystrophy is an expansion of a trinucleotide (CTG) in a gene on the long arm of chromosome 19 that codes for a protein kinase. The gene product was named myotonin-protein kinase, and it is thought to be involved in sodium- and chloride-channel

function. In successive generations, this repeating sequence has a tendency to increase, sometimes into the thousands (normal is <40 CTG repeats), and the extent of repetition correlates with the severity of the disease. Thus, each succeeding generation is likely to get more extensive manifestations and earlier presentations of the disease (i.e., the phenomenon of “anticipation”). This trinucleotide repeat phenomenon is also seen in Huntington disease and Fragile X syndrome.
201. How does the pathophysiology of infant botulism differ from that of food-borne and wound botulism?
• Infant botulism results from the ingestion of Clostridium botulinum spores that germinate, multiply, and produce toxin in the infant’s intestine. This is called a toxi-infection. The source of the spores is often unknown, but it has been linked to honey in some cases, and spores have been found in corn syrups. Therefore, these foods are not advised for infants younger than 1 year old.
• Food-borne botulism involves cases in which preformed toxin is already present in the food. Improper canning and anaerobic storage permit spore germination, growth, and toxin formation, which result in symptoms if the toxin is not destroyed by proper heating.
• Wound botulism occurs if spores enter a deep wound and germinate.
202. What is the earliest indication for intubation in an infant with botulism?
Intubation is indicated if there is a loss of protective airway reflexes. This occurs before respiratory compromise or failure because diaphragmatic function is not impaired until 90% to 95% of the synaptic receptors are occupied. An infant with hypercarbia or hypoxia is at very high risk for imminent respiratory failure.

Schreiner MS, Field E, Ruddy R: Infant botulism: a review of 12 years’ experience at the Children’s Hospital of Philadelphia,
Pediatrics 87:159–165, 1991.

203. In an infant with severe weakness and suspected botulism, why is the use of aminoglycosides relatively contraindicated?
The botulism toxin acts by irreversibly blocking acetylcholine release from the presynaptic nerve terminals. Aminoglycosides, tetracyclines, clindamycin, and trimethoprim also interfere with acetylcholine release; therefore, they have the potential to act synergistically with the botulinum toxin to worsen or prolong neuromuscular paralysis.
204. What are the two most common symptoms in children with juvenile myasthenia gravis? Ptosis and diplopia. Myasthenia gravis is characterized by a highly variable clinical course of fluctuating weakness (characteristically with increasing contractions) that initially involves muscles that are innervated by the cranial nerves. It is caused by a defect in neuromuscular transmission that is caused by an autoimmune antibody-mediated attack on the acetylcholine receptors.
205. What are the risks to a neonate who is born to a mother with myasthenia gravis? Passively acquired neonatal myasthenia develops in about 10% of infants born to myasthenic mothers because of the transplacental transfer of antibody directed against acetylcholine receptors (AChR) in striated muscle. Signs and symptoms of weakness typically arise within the first hours or days of life. Pathologic muscle fatigability commonly causes feeding difficulty, generalized weakness, hypotonia, and respiratory depression. Ptosis and impaired eye movements occur in only 15% of cases. The weakness virtually always resolves as the body burden of anti-AChR immunoglobulins diminishes. Symptoms typically persist for about 2 weeks but may require several months to disappear completely. General supportive treatment is usually adequate, but oral or intramuscular neostigmine may help diminish symptoms.
206. How does the pathophysiology of juvenile versus congenital myasthenia gravis differ?
JUVENILE (and adult) myasthenia GRAVIS is caused by circulating antibodies to the AChR of the postsynaptic neuromuscular junction. Occurrence is rare before the age of 2 years. Congenital myasthenia GRAVIS is a nonimmunologic process. It is caused by morphologic or physiologic features affecting the presynaptic and postsynaptic junctions, including defects in ACh synthesis, end-plate acetylcholinesterase deficiency, and end plate AChR deficiency. Neonatal myasthenia GRAVIS refers to the transient weakness that occurs in infants of mothers with myasthenia gravis.

207. How is the edrophonium (Tensilon) test done?
Edrophonium is a rapid-acting anticholinesterase drug of short duration that improves symptoms of myasthenia gravis by inhibiting the breakdown of ACh and increasing its concentration in the neuromuscular junction. A test dose of 0.015 mg/kg is given intravenously; if it is tolerated, the full dose of 0.15 mg/kg (up to 10 mg) is given. If measurable improvement in ocular muscle or extremity strength occurs, myasthenia gravis is likely. Because edrophonium may precipitate a cholinergic crisis (e.g., bradycardia, hypotension, vomiting, bronchospasm), atropine and resuscitation equipment should be available.
208. Does a negative antibody test exclude the diagnosis of juvenile myasthenia gravis? No. Up to 90% of children with juvenile myasthenia have measurable anti-AChR antibodies, but, in the other 10%, continued clinical suspicion is necessary because their symptoms are usually milder (e.g., ocular muscle weakness, minimal generalized weakness). In these children, other tests (e.g., edrophonium, electrophysiological studies, single-fiber electromyography) may be needed to make the diagnosis.

Della Marina A, Trippe H, Lutz S, Schara U: Juvenile myasthenia gravis: recommendations for diagnostic approaches and treatment, Neuropediatrics 45:75–83, 2014.

209. What are the four characteristic features of damage to the anterior horn cells?
Weakness, fasciculations, atrophy, and hyporeflexia.
210. What processes can damage the anterior horn cells?
• Degenerative (spinal muscular atrophy): Werdnig-Hoffmann, Kugelberg-Welander
• Metabolic: Tay-Sachs disease (hexosaminidase deficiency), Pompe disease, Batten disease (ceroid-lipofuscinosis), hyperglycinemia, neonatal adrenoleukodystrophy
• Infectious: Poliovirus, Coxsackie virus, echoviruses
211. What is the primary genetic abnormality in infants and children with spinal muscular atrophy (SMA)? Disruption of the survival motor neuron 1 (SMN1) gene. SMAs are a group of diseases that affect the motor neuron, resulting in widespread muscular denervation and atrophy. Incidence is estimated at 1 in 6000 to 10,000 newborns with a carrier frequency between 1 in 40 to 60. SMAs are the second most common hereditary neuromuscular disease after Duchenne muscular dystrophy. Extra copies of the SMN2 gene (a companion protein-coding gene) modify the clinical outcome. How changes in the SMN protein result in the disease process and phenotypic variability is unclear.

Nurputra DK, Lai PS, Harahap NI, et al: Spinal muscular atrophy: from gene discovery to clinical trials, Am Hum Genet
77:435–463, 2013.
Spinal Muscular Atrophy Association: www.smafoundation.org. Accessed March 6, 2015.

212. How are the inherited progressive spinal muscular atrophies distinguished?
See Table 13-10.
213. What are muscular dystrophies? A muscular dystrophy is an inheritable myopathy that affects limbs or facial muscles and is progressive, with pathologic evidence of degeneration or regeneration without any abnormal storage material.

Muscular Dystrophy Association: www.mdausa.org. Accessed on March 6, 2015.

214. What is the clinical importance of dystrophin? Dystrophin is a muscle protein that is presumed to be involved in anchoring the contractile apparatus of striated and cardiac muscle to the cell membrane. As a result of a gene mutation, this protein is completely missing in patients with Duchenne muscular dystrophy. On the other hand, muscle tissue from patients with Becker muscular dystrophy contains reduced amounts of dystrophin or, occasionally, a protein of abnormal size.
215. How are Duchenne and Becker muscular dystrophies distinguished?
See Table 13-11.

Table 13-10. Progressive Spinal Muscular Atrophies (SMAs)

DISORDER
INHERITANCE AGE OF ONSET
CLINICAL FEATURES
Acute infantile SMA (Werdnig-Hoffmann disease, SMA type 1) Autosomal recessive In utero to 6 mo Frog-leg posture; areflexia; tongue atrophy and fasciculations, progressive swallowing, and respiratory problems;
survival <4 yr
Intermediate SMA (chronic Werdnig- Hoffmann disease, SMA type 2) Autosomal recessive; rarely autosomal dominant 3 mo to 15 yr Proximal weakness; most sit unsupported; decreased or absent reflexes; high incidence of scoliosis, contractures; survival may be up to 30 yr
Kugelberg-Welander disease (SMA type 3) Autosomal recessive; rarely autosomal dominant 5-15 yr May be part of the spectrum of SMA 2; hip girdle weakness; calf hypertrophy; decreased or absent reflexes;
may be ambulatory until fourth decade
Adult-onset SMA
(SMA type 4) Autosomal recessive After age 30 years Mild to moderate muscle weakness, most commonly proximal; tremors, twitching; normal life
Adapted from Parke JT: Disorders of the anterior horn cell. In McMillan JA, DeAngelis CD, Feigin RD, Warshaw JB, editors:
Oski’s Pediatrics: Principles and Practice, ed 3. Philadelphia, 1999, JB Lippincott, p 1959.

Table 13-11. Duchenne Versus Becker Muscular Dystrophy
GENETICS DIAGNOSIS MANIFESTATIONS
Duchenne 1 in 3500 male births Whole-blood DNA may Clinically evident at 3-5 years of age X-linked reveal a deletion in Regular, stereotyped course Several different deletions, about 65%; otherwise, of progressive proximal
point mutations in electromyogram and weakness dystrophin gene result muscle biopsy studies Calf hypertrophy
in a completely are definitive Loss of ambulation by 9-12 years nonfunctional protein Worsening scoliosis and
New mutations occur contractures
Carrier females may have Eventual dilated cardiomyopathy
mild weakness or and/or respiratory failure
cardiomyopathy Life expectancy of 16-19 years
Becker 1 in 20,000 male births More benign clinical course Clinically evident during early second X-linked Reduced dystrophin levels decade
Various mutations in in muscle cells (by Milder, slower course as compared dystrophin gene result immunostaining) or with Duchenne
in reduced amount of abnormal dystrophin Calf pseudohypertrophy or partially functional Pes cavus
protein Cardiac and central nervous system
involvement unusual Ambulatory until 18 years or beyond Life expectancy twice as long as
compared with Duchenne
Adapted from Tsao VY, Mendell JR: The childhood muscular dystrophies: making order out of chaos. Semin Neurol 19:9–23, 1999.

216. What causes the calf hypertrophy seen in Duchenne muscular dystrophy?
Calf hypertrophy (Fig. 13-9) occurs primarily from replacement of muscle fibers with fat and fibrous tissue. When palpated, the calf has an unusually firm rubbery feel. Other muscles, including the tongue, can be enlarged with replacement tissues.

Figure 13-9. Calf enlargement in Duchenne muscular dystrophy. (From Perkin GD, Miller DC, Lane R, et al, editors: Atlas of Clinical Neurology, ed 3. Philadelphia, 2011, Saunders ELSEVIER, p 58.)

217. Is corticosteroid therapy effective for the treatment of Duchenne muscular dystrophy?
Several studies have documented an improvement in strength with an optimal dose of prednisone of
0.75 mg/kg per day. The strengthening effect lasts for up to 3 years while the steroid is continued. Despite mediation and supportive care, loss of ambulation, respiratory failure, and compromised cardiac function remain the inevitable outcomes. However, clinical trials are currently underway using gene therapy, which could eventually revolutionize the treatment of the disease.

Al-Zaidy S, Rodino-Klapac L, Mendell JR: Gene therapy for muscular dystrophy: moving the field forward, Pediatr Neurol
51:607–618, 2014.

218. What is the most likely diagnosis in a child with progressive walking difficulties evolving over several days?
Guillain-Barré syndrome (GBS) is an acute demyelinating neuropathy that is characterized by ascending, acute, progressive peripheral and cranial nerve dysfunction and paresthesias. In younger children (<6 years), it may be heralded by pain. It is frequently preceded by a viral respiratory or gastrointestinal illness. The disease is characterized by the presence of multifocal areas of the inflammatory demyelination of nerve roots and peripheral nerves. As a result of the loss of the healthy
myelin covering, the conduction of nerve impulses (action potentials) may be blocked or dispersed. Axonal injury may also occur. The resulting clinical effects are predominantly motor (i.e., the evolution of flaccid, areflexic paralysis). There is a variable degree of motor weakness. Some individuals have mild brief weakness, whereas fulminant paralysis occurs in others. Autonomic signs (e.g., tachycardia, hypertension) and sensory symptoms (e.g., painful dysesthesias) are not uncommon, but they are overshadowed by the motor signs. More than half of these patients develop facial involvement, and mechanical ventilation may be required.

Yuki N, Hartung H-P: Guillain-Barré syndrome, N Engl J Med 366:2294–2304, 2012.

219. What are subtypes of GBS?
Subtypes are classified based on the clinical picture, laboratory results (including antiglycoside autoantibody patterns) and results of electromyography (EMG) and nerve conduction velocity (NCV) studies, which differentiate the relative effect of damage from either demyelination or axonal injury. One commonly used classification is:
• AIDP: Acute Inflammatory Demyelinating Polyneuropathy. This is the primary demyelinating form. EMG/NCV studies show evidence of demyelination of both motor and sensory nerves. In North America and Europe, 90% of cases of GBS involve this type.
• AMSAN: Acute Motor-Sensory Axonal Neuropathy. The axon of the nerve is considered to be the primary target, with secondary loss of myelin.
• AMAN: Acute Motor Axonal Neuropathy. Motor nerves alone are affected without sensory loss. EMG does not show demyelination. Axonal variants (AMSAN and AMAN) are much more frequent in Asia, accounting for 40% to 50% of cases.
• Variants: Miller Fisher syndrome (MFS) is characterized by gait ataxia, areflexia, and ophthalmoparesis. Bickerstaff encephalitis is brainstem encephalitis with encephalopathy in addition to the features seen in MFS. Both are associated with autoantibodies to glycoside GQ1b.

Arcila-Londono X, Lewis RA: Guillain-Barré syndrome, Semin Neurol 32:179–132, 2012.

220. What CSF findings are characteristic of GBS?
The classic CSF finding is the albuminocytologic dissociation. Most common infections or inflammatory processes generate an elevation of white blood cell count and protein. The CSF profile in GBS includes a normal cell count with elevated protein, usually in the range of 50 to 100 mg/dL; however, at the onset of disease, the CSF protein concentration may be normal.
221. Outline the management of acute GBS.
Early clinical monitoring is focused on the development of bulbar or respiratory insufficiency. Bulbar weakness manifests as unilateral or bilateral facial weakness, diplopia, hoarseness, drooling, depressed gag reflex, or dysphagia. Frank respiratory insufficiency may be preceded by air hunger, dyspnea, or a soft muffled voice (hypophonia). The autonomic nervous system is occasionally involved, and this is signified by the presence of cardiac arrhythmia, labile blood pressure and body temperature, and urinary retention. The management of GBS includes the following:
• Observation in an intensive care unit is critical, with frequent monitoring of vital signs.
• The early institution of IVIG or plasmapheresis shortens the clinical course and lessens long-term morbidity; corticosteroid therapy is thought to be ineffective.
• If bulbar signs are present, the patient should receive nothing orally, and the mouth is suctioned frequently. Hydration is maintained intravenously, and nutritional support is provided by nasogastric feedings.
• The vital capacity (VC) is measured frequently. In children, the normal VC may be calculated as VC 200 mL age in years. If the VC falls below 25% of normal, endotracheal intubation is performed. Careful pulmonary toilet is conducted to minimize atelectasis, aspiration, and pneumonia.
• Meticulous nursing care includes careful patient positioning to prevent pressure sores, compression of peripheral nerves, and venous thrombosis.
• Physical therapy is conducted to prevent the development of contractures by passive range-of- movement exercises and splinting to maintain physiologic hand and limb postures until muscle strength returns.

Hughes RA, Swan AV, van Doorn PA: Intravenous immunoglobulin for Guillain-Barré syndrome, Cochrane Database Syst REV 9:CD002063, 2014.
Agrawal S, Peake D, Whitehouse WP: Management of children with Guillain-Barré syndrome, Arch Dis Child Educ Pract Ed 92:161–168, 2007.

222. What is the prognosis for children with GBS? Children appear to recover more quickly and more fully than adults. Most have good neurologic recovery, although approximately 20% to 40% may have some longer term residual symptoms (including paresthesias and fatigue). In children, the long-term outcome is not substantially different among the GBS subtypes. In rare cases, the neuropathy may recur as a chronic inflammatory demyelinating

polyneuropathy (CIDP). There is debate whether CIDP is a long-term continuation of AIDP or a separate illness with a different pathogenesis.

Roodbol J, de Wit MC, Aarsen FK, et al: Long-term outcome of Guillain-Barré in children, J Peripher NERV Syst 19:121– 126, 2014.
Vajsar J, Fehlings D, Stephens D: Long-term outcome in children with Guillain-Barré syndrome, J Pediatr 142:305– 309, 2003.
GGS/CIDP Foundation International: www.gbs-cidp.org. Accessed on March 6, 2015.

223. Does multiple sclerosis (MS) present during childhood?
Uncommonly. It is estimated that about 3% to 5% of patients with multiple sclerosis experience their first attack <18 years of age and onset <10 years is quite uncommon (<1%). Studies of affected children demonstrate a variable predominance of boys during early childhood and females during adolescence. Ataxia, muscle weakness, and transient visual or sensory symptoms are relatively
common presentations. CSF examination may demonstrate mild (<25 cells/mm3) mononuclear pleocytosis with an increasing probability of oligoclonal bands with each recurrence. MRI is the single most useful diagnostic test: the presence of multiple, periventricular white matter plaques (bright areas
on T2 images) confirms the diagnosis.

Bigi S, Banwell B: Pediatric multiple sclerosis, J Child Neurol 27:1378–1383, 2012.

SPINAL CORD DISORDERS
224. Which sacral dimples and coccygeal pits in a newborn are concerning for an occult spinal dysraphism (OSD)?
These occur in up to 4% of newborns. Isolated simple sacral dimples are rarely associated with a significant spinal abnormality. However, certain features are more likely to be associated with an OSD (such as tethered cord syndrome) and warrant a screening ultrasound.
• Location above the gluteal crease (typically >2.5 cm from the anus)
• Deep dimples (if base cannot be visualized, do not probe because of risk for introducing an
infection if a direct communication with the spinal canal is present)
• Larger size (>0.5 cm)
• Pits with cutaneous markers (lipoma, hypertrichosis, hemangioma)

Kucera JN, Coley I, O’Hara S, et al: The simple sacral dimple: diagnostic yield of ultrasound in neonates, Pediatr Radiol
45:211–216, 2015.
Williams H: Spinal sinuses, dimples, pits and patches: what lies beneath? Arch Dis Child Educ Pract Ed 91:75–80, 2006.

KEY POINTS: NEONATAL SACRAL FINDINGS SUGGESTIVE OF OCCULT SPINAL DYSRAPHISM
1. Location above the gluteal crease (typically >2.5 cm from the anus)
2. Deep dimples
3. Larger dimple size (>0.5 cm)
4. Sacral pits with cutaneous markers (lipoma, hypertrichosis, hemangioma)

225. What are the two main features of the Chiari malformations?
Cerebellar elongation and protrusion of the foramen magnum into the cervical spinal cord. Anatomic anomalies of the hindbrain and skeletal structure result in different positioning of the various structures relative to the upper cervical canal and foramen magnum with different clinical features.
226. What are the types of Chiari malformations?
Type I is the most common, but clinically the least severe and is generally asymptomatic during childhood. It is often diagnosed as an incidental finding on cervical MRI scans for neck pain and/or headache. The presentation of a Chiari I malformation may be insidious. There may be paroxysmal

vertigo, drop attacks, vague dizziness, and headache, which may be increased by the Valsalva maneuver. Occipital headache precipitated by exertion may progress to torticollis, down-gaze nystagmus, periodic nystagmus, and oscillopsia (objects in the visual field oscillate). MRI findings include malformations of the base of the skull and of the upper cervical spine, including hydromyelia and syringomyelia, which is a cyst (or syrinx) in the spinal cord that can expand and
elongate over time. Surgical treatment is typically reserved only for symptomatic patients or those with a syrinx.
Type II is the so-called “classic” Chiari malformation (known historically as Arnold-Chiari malformation). Medulla and cerebellum, together with part or the entire fourth ventricle, are displaced into the spinal canal (Fig. 13-10). A variety of cerebellar, brainstem, and cortical defects can occur. This type is strongly associated with noncommunicating hydrocephalus and lumbosacral myelomeningocele.
Type III comprises any of the features of types I and II, but the entire cerebellum is herniated throughout the foramen magnum, with a cervical spina bifida cystica. Hydrocephalus is a common feature.

Baisden J: Controversies in Chiari I malformations, Surg Neurol Int 3(Suppl 3):S232–237, 2012.

Figure 13-10. A midsagittal T1-weighted MRI of patient with type II Chiari malformation. The cerebellar tonsils (white arrow) have descended below the foramen magnum (black arrow). Note the small, slitlike fourth ventricle, which has been pulled into a vertical position. (From Kleigman RM, Stanton BF, Schor NF, et al, editors: Nelson Textbook of Pediatrics, ed 19. Philadelphia, 2011, ELSEVIER
Saunders, p 2009).

KEY POINTS: EARLY CLUES TO SPINAL CORD COMPRESSION
1. Scoliosis producing sustained poor posture
2. Back or abdominal pain beginning abruptly during sleep
3. Increased sensitivity of spinal column to local pressure or percussion
4. Bowel or bladder dysfunction
5. Diminished sensation in the anogenital region and lower limbs

227. What are types of spina bifida?
Spina bifida refers to malformations that result from failure of closure at the caudal end of the neural tube, as well as the overlying vertebral arches during embryogenesis. This can range from an asymptomatic defect such as spina bifida occulta in which the two halves of the vertebral arch fail to close to increasing displacement of the spinal cord (myelomeningocele) to the most severe form, myeloschisis, with exposed nervous tissue surrounded by no membrane (Fig. 13-11).

DIFFERENT TYPES OF SPINA BIFIDA

Spinal cord
Dura matter
and arachnoid

Subarachnoid space

Neural arch

Meningocele

Subarachnoid space

Displaced spinal cord
and nerve roots

Neural plate

Subarachnoid space

Spinal ganglia and nerve roots

Meningomyelocele Myeloschisis
Figure 13-11. The different types of spinal bifida. (From Perkin GD, Miller DC, Lane R, et al, editors: Atlas of Clinical Neurology, ed 3. Philadelphia, 2011, Saunders ELSEVIER, p 267.)

228. How common are asymptomatic spinal anomalies in normal children?
Up to 5% of children have spina bifida occulta, an incomplete fusion of the posterior vertebral arches, which is usually noted as an incidental radiographic finding. The defect most commonly involves the lower lumbar lamina of L5 and S1.
229. What is the full anatomic expression of myelomeningocele?
• The presence of unfused or excessively separated vertebral arches of the bony spine (spina bifida)
• Cystic dilation of the meninges that surround the spinal cord (meningocele)
• Cystic dilation of the spinal cord itself (myelocele)
• Hydrocephalus and the spectrum of congenital cerebral abnormalities
230. What is the likelihood that a patient with myelomeningocele will have hydrocephalus?
Hydrocephalus is seen in 95% of children with thoracic or high lumbar myelomeningocele. The incidence decreases progressively with more caudal spinal defects to a minimum of 60% if the myelomeningocele is located in the sacrum.
231. What is the usual cause of stridor in a child with myelomeningocele? The stridor is usually caused by dysfunction of the vagus nerve, which innervates the muscles of the vocal cords. In their resting position, the edges of the cords meet in the midline; during speech, they move apart. Hence, in bilateral palsies of the vagus nerve, the free edges of the vocal cords are closely opposed and obstruct air flow, thereby resulting in stridor. In symptomatic patients, the motor nucleus of the vagus nerve may be congenitally hypoplastic or aplastic. More commonly, the vagal dysfunction is believed to arise from a mechanical traction injury caused by hydrocephalus, which produces

progressive herniation and inferior displacement of the abnormal hindbrain. Shunting the hydrocephalus may alleviate the traction and improve the stridor. Sometimes the later recurrence of stridor indicates the reaccumulation of hydrocephalus as a result of ventriculoperitoneal shunt failure.
232. What are the principal options for managing urinary incontinence in patients with myelomeningocele?
About 80% of patients have a neurogenic bladder, which most commonly manifests as a small, poorly compliant bladder and an open and fixed sphincter. Options include the following:
• Clean intermittent catheterization, which results in more complete emptying than simple Credé maneuvers
• Artificial urinary sphincter to increase outlet resistance
• Surgical urinary diversion (e.g., suprapubic vesicostomy), which is uncommonly used
• Augmentation cystoplasty to increase bladder capacity in combination with the use of oxybutynin (a smooth muscle antispasmodic)

Mourtzinos A, Stoffel JT: Management goals for the spina bifida neurogenic bladder: a review from infancy to adulthood,
Urol Clin North Am 37:527–535, 2010.

233. How frequently is myelomeningocele associated with intellectual disability? Only 15% to 20% of patients have associated intellectual disability (mental retardation). Hydrocephalus per se does not cause the mental retardation that is associated with this syndrome. Children with appropriately treated congenital hydrocephalus caused by simple aqueductal stenosis usually have normal psychomotor development. Only severe hydrocephalus with a very thick cortical mantle predicts lower intelligence. Intellectual disability is usually attributed to acquired secondary CNS infection or subtle microscopic anomalies of neuronal migration and differentiation, which may coexist with the macroscopically visible malformation of the hindbrain.
234. In an infant born with myelomeningocele, how does the initial evaluation predict long-term ambulation potential?
The level of motor function—and not the level of the defect—is most predictive of ambulation.
• Thoracic: No hip flexion is noted. Almost no younger children will ambulate, and only about one-third of adolescents will ambulate with the aid of extensive braces and crutches.
• High lumbar (L1, L2): The patient is able to flex the hips, but there is no knee extension. About one-third of children and adolescents will ambulate, but only with extensive assistive devices.
• Mid-lumbar (L3): The patient is able to flex the hips and extend the knee. The percentage of those able to ambulate is midway between those with high and low lumbar lesions.
• Low lumbar (L4, L5): The patient is able to flex the knee and dorsiflex the ankle. Nearly half of younger children and nearly all adolescents will ambulate, with varying degrees of braces or crutches.
• Sacral (S1-S4): The patient is able to plantar flex the ankles and move the toes. Nearly all children and adolescents will ambulate, with minimal or no assistive devices.
Acknowledgment
The editors and the author gratefully acknowledge contributions by Drs. Kent R. Kelly, Douglas R. Nordli, Jr., Peter Bingham, and Robert R. Clancy that were retained from the first five editions of Pediatric Secrets.